quinta-feira, 13 de fevereiro de 2014

Se não acredita em Deus, como acha que vinhemos parar aqui na terra?‎

Evoluímos a partir de espécies precedentes que, por sua vez, evoluíram de outras até o primeiro ser vivo que surgiu, por acaso, a partir da matéria inanimada. Simples assim.

Então ser cético é não ter certeza de nada? Mas viver assim; sem uma certeza é tão vago.‎

Não é bem assim. E sempre estar aberto a considerar que se esteja errado. Ou seja, é ter uma certeza, quando se tem, sempre provisória. Você pode considerar que sabe algo e agir com base nesse conhecimento. Mas tem que estar consciente que seu conhecimento pode não estar correto e se dispor a mudar o modo de pensar toda vez que for convencido disso por bons argumentos. Há casos, contudo, que se pode quase estar seguro do conhecimento. Por exemplo, de que o Sol existe. Mas, principalmente nos casos em que o conhecimento não seja verificado por evidências diretas, mas sim por comprovações indiretas, a posição correta é a de permanecer com o juízo em suspenso sobre a veracidade do conhecimento. Não acho que isso seja vago. É o mais sensato a ser feito e não vejo problema nenhum em agir com base em suposições de certezas não garantidas. Em verdade, não vejo outra alternativa.

Existem pessoas mais inteligentes que as outras ou com esforço todo mundo pode ter a mesma capacidade intelectual?‎

Existem. A inteligência, em parte é genética, em parte adquirida. A genética fornece uma maior ou menor capacidade de desenvolver a inteligência, com menos ou mais esforço. Mas ela é desenvolvida pelo enfrentamento de desafios, especialmente na tenra infância. Por isso é importantíssimo que os pais propiciem aos bebês, sempre, estímulos ao desenvolvimento de sua inteligência. Que sempre estejam vendo muitas coisas, examinando, tocando, lambendo, cheirando tudo. Engatinhando para todo lado e passeando para perceber o mundo. Conversando com muitas pessoas, que não fiquem tatibitateando, mas falando normal. Que brinquem com brinquedos estimulantes e não aqueles de pilha que funcionam sozinhos. A criança é que tem que inventar a brincadeira. Mesmo tendo deixado de ser criança é possível estimular e aprimorar a inteligência com desafios sensoriais, motores, cognitivos e outros. Facilitar emburrece. Mas quem tem uma aptidão genética consegue esse aprimoramento mais fácil e rapidamente. Não é todo mundo que vai conseguir atingir o mesmo nível máximo de inteligência. Isso é como a aptidão física e a beleza. Há quem seja fraco ou feio de nascença mesmo, como há quem seja burro. Mas pode deixar de ser um pouco, com exercícios. A escola precisa se dedicar a isso, mas parece que não se importa. Só importa com a aquisição de conhecimentos e algumas habilidades. Mas não com o aprimoramento da inteligência e, tampouco, do caráter. Isso teria que ser uma exigência curricular, com aferição por meio de avaliações periódicas, do progresso da inteligência e da adoção de valores éticos

Professor, na sua opinião, em algumas décadas a tecnologia e a medicina (e suas vertentes) ainda pode evoluir, descobrir e criar coisas que hoje é inimaginável ou improvável para a maioria?‎

Claro que sim. É só ver que grande parte do que existe atualmente, em termos de tecnologia, nem era cogitado que pudesse existir há, digamos, duzentos anos atrás. Do mesmo modo, nem imaginamos o que poderá existir daqui a duzentos anos. Quando se vê o que o pessoal imaginava que seria o mundo atual, há tempos atrás, pode-se perceber o quanto essa previsão está ligada ao que existe no momento. Do mesmo modo, se imaginarmos o mundo daqui a cem anos, o faremos com base no que existe hoje. E pode ser totalmente diferente.
http://www.inspi.com.br/2013/11/mundo-no-ano-2000/

é possível afirmar que o universo não existiria se nós não existissimos? Pois não haveria vida inteligente pra observá-lo (considerando que sejamos a única vida inteligente no universo)

Claro que não. O Universo não tem nada a ver com a existência de seres inteligentes para observá-lo. Aliás, a maior parte do tempo, ele existiu sem que nenhum ser inteligente o observasse. Nem mesmo não inteligente. E poderia continuar existindo sem que houvesse surgido vida em lugar nenhum e, portanto, inteligência. Não é verdade que ser é ser percebido, como o queria Berkeley. Tudo o que é, é o que é, seja ou não percebido. Simplesmente, se não é percebido, não se sabe que existe ou não. Mas se nada que não é percebido não existisse, nunca se descobririam coisas novas.

A natureza é simétrica ?‎

Em alguns assuntos sim, em outros não. Não há uma simetria total e nem uma assimetria total. As leis de conservação decorrem de certas simetrias. Mas nem tudo na natureza obedece leis de conservação, como a entropia, por exemplo. Por isso é que, se houver Big Crunch, ele não retornará à mesma situação do Big Bang, pois a entropia terá aumentado.

Professor, qual melhor maneira de entender as ideias de um filosofo, ler livros dele ou de terceiros sobre ele? Por exemplo, melhor ler os diálogos de Platão ou ler alguém que escreveu sobre ele?‎

Eu recomendo que se leia o que se disse sobre ele para começar e, depois, o que ele mesmo disse. Acho assim melhor do que o contrário. Muitos, contudo, discordam de mim. Esse método, inclusive, é melhor para se selecionar o que ler de cada um, uma vez que é difícil ler tudo de todo mundo. Inclusive sugiro a leitura de compêndios gerais de Filosofia e de História da Filosofia antes de ler obras a respeito de tal ou qual filósofo. Por fim, leiam-se as próprias obras dos filósofos.

A tração gravitacional de uma estrela é exatamente um quarto da de uma estrela semelhante a metade da distância .Ernesto me explique isto

Isso é o que diz a Lei da Gravitação Universal, descoberta por Newton, que descreve a intensidade da interação gravitacional entre dois corpos como sendo a mesma em cada um (mesmo que tenham massas diferentes), atrativa e ao longo da reta que une seus centros de massa, de valor proporcional ao produto das massas e inversamente proporcional ao quadrado da distância entre os centros. Se a distância dobrar, a intensidade se torna um quarto, Se triplicar, se torna um nono e assim por diante. Se for metade, se torna o quádruplo. Isso é que significa ser proporcional ao inverso do quadrado da distância. Isto é, o produto da intensidade da interação gravitacional pela distância ao quadrado é constante para um dado par de corpos

Qual curso o senhor acha melhor, matemática ou física? Tendo em vista querer ser professora de universidade.‎

Apesar de ter me graduado em Matemática, eu prefiro Física. Acho mais interessante, além do que, como físico, pode-se saber muita matemática também. Mas a Física tem o atrativo de investigar a própria essência profunda da realidade, pois, afinal, tudo se reduz à Física, em última análise. Até o amor, a economia e a política.

Professor, suponha a existência de um 'planeta toroidal', existe alguma condição da massa, densidade ou velocidade na qual esse planeta se sustente ou inevitavelmente colapsará por sua própria gravidade? Se sim, é possível imaginar alguma condição no universo que gerasse um planeta assim?

A forma de um planeta e de uma estrela é gerada pelo concurso da gravidade e da inércia. Isso sempre levará à formação de um esferoide prolato, isto é, uma esfera achatada nos polos. Nunca levará á um toro. Pode levar à formação de anéis de partículas, mas só se houver um planeta ou estrela no centro. Jamais se formará um anel sem nada dentro em torno de que ele orbite.

O que é arqueologia?

Trata-se do estudo do legado do homem antigo, De suas construções, de sua arte. É um estudo de história com base não em documentos mas em objetos, edificações, obras de arte, túmulos, cidades e outros achados reveladores do modo de vida e dos feitos das culturas e civilizações mais antigas

Professor, em uma de suas respostas você cita "Todavia, locomover-se a velocidade iguais ou maiores que a da luz é uma verdadeira impossibilidade, não importa que tecnologia se tenha." - Lhe pergunto, por que? Nem com uma futura tecnologia bem distante?

Porque não depende da tecnologia. É uma impossibilidade da natureza. Imprimir a qualquer corpo que possua massa uma velocidade igual à da luz requer um suprimento infinito de energia. Isso não é possível, não importa qual seja a tecnologia.

''Só os estúpidos precisam de organização. Os gênios controlam o caos.'' Albert Einstein - Achei interessante. Sabe se é verdadeiro?‎

Não sei se, de fato, Einstein disse isso. Mas concordo. Todavia não chego aos extremos de estúpidos e gênios. Pessoas mais inteligentes, certamente, sabem se orientar no caos e se safam bem sem muita organização. Organizar requer grande dispêndio de tempo e energia. Quem conseguir fazer tudo a contento, sem organização, pode dispensá-la. Sinceramente, há tempos eu gostava de ser organizado. Depois vi que era bobagem. Não ligo para uma certa bagunça. Meu escritório é assim. Mas eu sei onde cada coisa se encontra e acho. Mesmo meu modo de raciocinar não é organizado. Cuido de muitos assuntos ao mesmo tempo. Para mim, não é problema nenhum.

Azul é a cor mais quente?‎

Não. dentro do espectro visível a mais quente é o roxo e a mais fria o vermelho. Em geral, contudo, não se tem uma cor pura, ficando uma chama mais quente misturada com as cores adjacentes, no caso, o azul. Apesar da temperatura das luzes roxa, azul e verde serem maiores do que as das cores amarela, laranja e vermelha, a sensação psicológica que transmitem é ao contrário. Roxo, azul e verde passam uma percepção de coisa mais fria, enquanto amarelo, laranja e vermelho a de coisa mais quente.

O que preciso fazer pra ser cosmologista?

Cursar a graduação (bacharelado de preferência) em Física ou Matemática e, então, o mestrado e o doutorado em Cosmologia, que, em geral, é uma área do mestrado e do doutorado em Física.

O dinheiro pedido pela igreja não é pedido apenas para pagar o salario do pastor.A igreja também tem gastos.Tem agua, luz, aluguel . alimentação e etc.Eu não sou evangélico e sou totalmente contra esses bandidos que se dizem pastor mas sem doação não tem como manter a igreja.Quem vai bancar ?

Doações voluntárias. Mas não cobrança por realização de batizados ou casamentos. Acho que cada igreja deveria ter um conselho de curadores que administraria suas finanças e não o padre ou o bispo. Completamente democrática e não autocrática. Os padres, pastores e bispos não podem receber nada pelo que fazem. Isso macula sua função religiosa. Têm que trabalhar de graça.

Professor,os únicos campos de pesquisa na física estão relacionado apenas com a mecânica quântica? A mecânica clássica já está completa?‎

A maior parte das pesquisas em Física atual se focam na estrutura da matéria, que é quântica, nas teorias quânticas de campo, na Relatividade Geral e na ótica quântica (lasers, por exemplo). A mecânica clássica está, mesmo, praticamente esgotada, em termos de pesquisa básica. Ela é usada em pesquisas de engenharia, especialmente nas relativas a escoamento de fluidos e termodinâmica. Também são feitas muitas pesquisas em Física Estatística e Sistemas Dinâmicos (caos), estes sim, inclusive com abordagem clássica, para estudo de fenômenos meteorológicos, por exemplo.

O que você acha da EPCAR ?

Excelente escola. Já lecionei lá de 1968 a 1976. Não sei se continua tão boa quanto era, mas a grande vantagem que via nela é que não dava a menor importância para o que iria cair nos vestibulares mas sim no que fosse preciso aprender para a vida profissional futura do oficial aviador. Na Física, por exemplo, não se estudava muita coisa irrelevante, como associação de resistores e capacitores e equilíbrio de barras. Em compensação se estudava momento angular, relatividade restrita, física quântica, física atômica, física nuclear, física estatística, lei de Gauss do Eletromagnetismo, ondas eletromagnéticas e muitos temas de relevância para a compreensão da estrutura do Universo que não são vistos, ainda, no Ensino Médio. Eram oito horas de aula por dia, com educação física todo dia e instrução militar uma vez por semana, além de horário obrigatório e vigiado de estudo diário, das 19 às 22 horas. Muito puxado mesmo. Bem exigente. Com desligamento por reprovação. Mas vale a pena, mesmo que não se vá ser oficial aviador. Se quiser fazer vestibular, que faça um cursinho depois. Mas o excelente ensino médio que vai se ter vale muito mais a pena do que ver tópicos que caem no vestibular.

Aquiles corre 10x mais rápido que uma tartaruga e dá ao animal 100m de vantagem. Ele corre 100 m e a tartaruga está 10 m à frente. Ele corre 10 m e a tartaruga está 1 m à frente. Corre 1 m, e a tartaruga ainda está 10 cm à frente e assim por diante ad infinitum de modo que nunca alcança a tartaruga‎

Essa é uma questão de convergência de uma série infinita de termos cada vez menores. Cada nova etapa dessas gasta um décimo do tempo da anterior, de modo que o tempo tenderá a zero rapidamente, quando, então, Aquiles alcançará a tartaruga e a ultrapassará. O paradoxo está em supor que esses tempos sejam sempre finitos, mas não são. Eles se tornam infinitesimais o que faz com que a ultrapassagem ocorra. Essa é a grande diferença entre a álgebra e a análise. E esse problema é um problema de análise e não de álgebra.

Professor,por que as leis da mecânica clássica não se aplica a mecânica quântica? Se tudo é formado por átomos,por que não percebemos as "loucuras" quântica no mundo macroscópico?

Por causa da estatística. A causalidade, por exemplo, emerge da incausalidade probabilistica sempre que se tiver o concurso de um grande número de eventos microscópicos para dar conta de um evento macroscópico. Força, pressão, temperatura, volume, posição, velocidade, energia e outros parâmetros dos sistemas de muitas partículas (acima de umas 10^20) são valores médios de valores individuais dos átomos. Mesmo que cada átomo possa ter uma velocidade qualquer e uns com os outros, uma força de interação qualquer, o resultado estatístico do conjunto convergirá para um valor estável, pois há compensações nas diversas flutuações. Mesmo que nunca se saiba quando um átomo particular emitirá um fótom quando excitado, pode-se saber que haverá alguma emissão de fótons e, até, quantos serão emitidos por segundo para provocar uma emissão de luz por um filamento aquecido, por exemplo.

Comentário sobre o aspecto informacional-holográfico da consciência

Dizer que a consciência, e mesmo toda a mente, seja holográfica é uma analogia com o fenômeno ótico, mas não completamente aderente. Na holografia ótica, cada trecho do holograma contém informação sobre a imagem completa. No caso mental um registro de memória está espalhado pelo cérebro todo, conforme o aspecto dessa memória (visual, olfativo, auditivo, tátil etc) que, ao ser recuperado, solicita a contribuição de todos esses lugares. Mas a informação não está completa em cada um deles. No caso da consciência, ela é uma espécie de percepção que o cérebro tem de seu próprio funcionamento, do mesmo modo que a auto-consciência é uma percepção que o cérebro tem do organismo todo como algo distinto do resto do universo, isto é, o "eu". Toda percepção envolve memorização, senão o cérebro não tem a continuidade da percepção. Isso é nítido na audição. Você só tem compreensão do que ouve (uma música, por exemplo), à medida que vai cotejando o que acabou de ouvir com o que está ouvindo agora. Se você só tivesse a percepção do momento presente, não entenderia o que está ouvindo. Então, a consciência, que é formada de múltiplas percepções, está espalhada pelo cérebro todo. E, de fato, trata-se de uma informação. Mas é preciso entender bem que nenhuma informação existe sem o seu registro físico. O que é, conceitualmente, uma informação, só tem existência se o seu conteúdo estiver registrado de alguma forma. No caso mental esses registros se dão pelo estabelecimento de conexões sinápticas. Elas são as marcas físicas (no caso, biológicas, mas biológicas são físicas) de tudo o que o cérebro registra. E o processamento mental, isto é, os registros e evocações de memórias, os juízos, os raciocínios, as emoções, os sentimentos, as intuições e tudo o mais, são processos de trocas de informações entre registros de memórias e de estabelecimento e desconstrução desses registros. Isso é a vida psíquica e é objeto de muito estudo neurológico, para decifrar completamente a correspondência entre um estado psíquico e um estado fisiológico do cérebro que lhe corresponda. Quando isso estiver estabelecido, se poderá escanear o cérebro e extrair o conteúdo dos pensamentos, raciocínios, sentimentos, emoções e tudo o mais. Mas ainda demora para se chegar lá.

A ciência é cética?‎

Certamente que sim. Não pode deixar de ser, senão não é ciência, pois esta é a busca sistematizada e justificada do conhecimento. E toda justificação é provisória, nunca sendo garantida. É aceita até que seja derrubada. Por isso a ciência precisa, sempre, duvidar de si mesma, justamente para envidar esforços continuados de justificação de suas assertivas, que têm que ser mudadas sempre que reveladas incorretas. Assim cresce a ciência e isso é sua maior qualidade: não ter certeza do que diz. A ciência é sempre aberta a todas as ponderações e contestações e não se importa de ser vencida e ter que se retratar, mudando seu pensamento. Jamais é dogmática.

Ernesto, uma vez uma pessoa me disse que ninguém definiu a filosofia tão bem quanto Hegel e que todo o resto foi construído a partir dele. Uns filósofos o apoiaram e outros o confrontaram. De todo o modo, ele foi o principal. Não conheço muito sobre ele, por isto, venho pedir tua opinião a respeito

Não concordo com isso. Não acho que Hegel tenha sido tão importante assim. Muito menos do que Kant, por exemplo. Ou Descartes, Spinoza, Locke, Hume. O idealismo de Hegel, para mim, é totalmente furado.

professor, porque quanto mais alto for o apartamento, mais intenso, pelo menos aparentemente, o barulho que acontece no térreo ? por exemplo, uma ambulância passa, ai lá no vigésimo andar dá para escutar ainda.. deveria não se escutar, eu achava.

O que eu posso dizer é que os sons mais agudos têm maior penetração. Assim, próximo do chão, os demais sons, mas graves, se misturam aos agudos e este fica menos distinto. Nos andares mais altos, os graves não chegam, pois são absorvidos, mas os agudos chegam e sozinhos, parecendo, pois, mais nítidos.

Você considera a posição ateísta a correta. Porém você disse que todos os seus filhos acreditavam em Deus. Você acha que você cometeu algum erro na educação deles pra eles não terem seguido a opção correta?

Não, pois eu os eduquei para serem livres pensadores e para crerem ou não no que bem entendessem. Jamais impingi meu ponto de vista. Só o expus.

Então o amor é uma ilusão, um artifício evolutivo? Parece tão bom, mas pode ferir bastante se não correspondido

Não é uma ilusão, nem um artifício. Ele provém do sexo, mas se sublimou no sentimento de querer bem, de proteção, como eu disse nas várias postagens que listei. Mas o amor não correspondido só fere se a pessoa tiver a visão da necessidade da correspondência e, especialmente, da exclusividade. Amor não é o que causa o sofrimento. O que causa é o desejo de ser amado sem que o seja. Mas a pessoa pode se convencer e mudar sua cosmovisão a respeito dos relacionamentos humanos, de modo a considerar que amar é bom, mesmo sem ser correspondido e a aceitar a pluralidade amorosa como algo perfeitamente normal e não se sentir menosprezada e nem menos querida se quem a ama não amar exclusivamente a ela. Esse modo de encarar o amor é muito mais benéfico para a disseminação da felicidade do que a concepção egoísta e exclusivista do amor. Na concepção altruísta, não há lugar para nenhum ciúme, nenhum controle e nenhuma posse do ser amado. A pessoa que ama de verdade que o ser amado completamente livre para amá-la ou não. E, não sendo amada em retribuição, nem por isso deixa de amar, pois o amor vale por si mesmo, independentemente de ser retribuído. Deixar de amar por não ser retribuído é egoísmo e mostra que, de fato, não se ama. Mesmo não sendo cristão, concordo com o mandamento de Jesus: "Amai-vos uns aos outros como eu vos amei". Note que nessa prescrição não há "exceto" nenhum. Nem é como a antiga prescrição judaica de "amar ao próximo como a si mesmo". É muito mais do que isso. É amar como Jesus amou e, para quem crê nisso, ele tinha um amor perfeito, pois era Deus.

uma pessoa pode viver 200 anos?‎

Acho que a atual configuração genética de nossa espécie, mesmo que a pessoa leve a vida toda de forma completamente saudável e nunca tenha sido acometida por doença nenhuma, não consegue passar de uns 125 anos. Isso é um problema de degenerescência dos telômeros.
http://pt.wikipedia.org/wiki/Tel%C3%B4mero

o que vc acha desse video ? Relacionado a religião: http://www.youtube.com/watch?v=XGM6qJ0wZ_M‎

Isso é um absurdo. Edir Macedo e outros líderes de igrejas neopentescostais, são como esses políticos corruptos. Ladrões mesmo. E enganadores do povo. Precisam ir para a cadeia sem sursis. Não consigo entender como o governo permite isso. Mas outras igrejas, até a católica, também exploram os fiéis. Para mim todo serviço religioso teria que ser gratuito e padres e pastores não ganharem nada por seu trabalho. Teriam outros empregos para se sustentar. O trabalho religioso teria que ser um voluntariado. Tudo nas igrejas teria que ser inteiramente de graça. Assim é que eu entendo.

Professor, fui a uma palestra sobre anarquismo, e sinceramente, é algo tão utópico que as vezes se torna difícil visualizar a humanidade seguindo esse modelo. Você, por ser tão inteligente, ainda acredita q um dia esse sistema dará certo? Pq? Será que o homem saberia se virar sozinho sem líderes?‎

Não é tão utópico não. É uma questão de educação. Claro que para se chegar lá demorarão, ainda, centenas de anos. Mas é o que tem acontecido com a humanidade nos últimos milhares de anos. A evolução tem sido no sentido cada vez mais democratizante, que é um passo para a anarquia. Aos poucos as tendências predatórias e dominadoras serão substituídas por atitudes colaborativas e compartilhantes, sem necessidade de obediência a ninguém para se fazer o que é certo. Para abreviar esse tempo é preciso que as pessoas conscientes façam um esforço educativo nesse sentido, não só pela propaganda da anarquia, mas pelo exemplo de atitudes anárquicas, como trabalhar de graça pelo bem dos outros. Note que anarquia, diferentemente do que se pensa, não é bagunça nenhuma. Pelo contrário, é uma situação extremamente ordeira, pois a ordem é assumida por vontade própria de cada um, sem imposição nenhuma. Não há polícia porque não há crime e não há crime porque ninguém quer cometê-lo.

Os humanos com a nossa tecnologia ainda vão conseguir viver tranquilamente na próxima era do gelo?

Alguns sim, mas todos não. Porque, possivelmente, a nova era do gelo começará em uns três mil anos e, até lá, acho que a humanidade ainda não se terá tornado totalmente rica, isto é, com a população inteiramente constituída de pessoas ricas, sem exceção. Ou que se tenha tornado inteiramente anárquica, sem dinheiro, mas com recursos para propiciar as condições de sobrevivência para todos. Assim, acho que a maioria morrerá de frio. Mas nem todos. Devem sobrar uns 15% que conseguirão sobreviver. Como as eras glaciais duram uns noventa mil anos, com uns dez mil de período interglacial, possivelmente nesse tempo a tecnologia evoluirá para permitir a expansão da humanidade no frio.
http://pt.wikipedia.org/wiki/Era_do_gelo
http://pt.wikipedia.org/wiki/Glacia%C3%A7%C3%A3o

Certa vez uma garota conhecida minha falou "eu fico mesmo com caras que tem namorada, e não estou nem aí, quem tem que cuidar do namoro é a menina, a culpa é dela de não cuidar e de confiar nele". Opinião sobre?‎

Em meu entendimento, todo relacionamento tem que envolver confiança sim. Total. Se a pessoa não for merecedora, que se encerre a relação. Mas a atitude de ficar com uma pessoa que tem algum compromisso de exclusividade relacional não é ética e revela um mal caratismo fundamental. Todavia não acho que seja preciso se firmar um compromisso de exclusividade nos relacionamentos. Um relacionamento pode ser completamente satisfatório e propiciar a plena realização amorosa, sexual e de amizade sendo totalmente aberto e livre, isto é, não exclusivista. Só que isso tem que ser do conhecimento e do consentimento de todos os envolvidos, que podem, mesmo, serem grandes amigos. O que estou falando é que uma menina pode ter mais de um namorado e um menino mais de uma namorada. O mesmo digo a respeito de esposos e esposas. Note que estou dizendo que pode e não que tem

Professor, quanto tempo a vida macroscópica ainda tem pela frente, em nosso planeta? Antes que as condições aqui se tornem muito agressivas

Mais uns três ou quatro bilhões de anos. Então o Sol se transformará em uma estrela gigante vermelha e evaporará toda a água da Terra, inviabilizando qualquer tipo de vida. Depois ele deve explodir e triturar ou evaporar o planeta todo, transformando-o em gás e poeira.

A espécie humana um dia vai se extinta, existiram novos humanos, em outro planeta com sorte, ou provimente de outro big bang?‎

É certo que a humanidade se extinguirá, pois o próprio universo não será eternamente propício à vida em lugar nenhum. Mas, muito antes disso, creio que em dez ou vinte milhões de anos, a humanidade não estará mais presente neste planeta. Muito provavelmente, até lá, outras espécies evoluirão e nos substituirão como as inteligentes daqui. Ou nós ou essas vindouras, colonizaremos outros planetas propícios à nossa vida. A própria Terra deixará de ser propícia a qualquer vida dentro de uns três ou quatro bilhões de anos, devido ao aquecimento do Sol que evaporará toda a água existente. Pelas mais atuais informações sobre a expansão do Universo, não haverá outro Big Bang. Este Universo se expandirá indefinidamente, tornando-se cada vez mais frio, até que não haja mais estrela nenhuma e, até mesmo, a matéria se desfaça pelo esgarçamento do espaço que destruirá as partículas sub-atômicas. Mas isso ainda demora dezenas ou centenas de trilhões de anos.

Professor, sei que talvez isto não tenha nada haver com tua área, porém, com sua experiência e bagagem de conhecimento você pode ter algo à dizer. O que se fazer caso o amado(a) não lhe corresponda o amor?‎

Continuar a amá-lo sem correspondência e, paralelamente, se abrir para a possibilidade de amar a outra pessoa também, sem precisar deixar de amar à primeira. Amor não exige exclusividade. Mas essa segunda pessoa tem que saber que você também ama à primeira. Do mesmo modo que você tem que aceitar que quem te ame possa amar, também, a outrem. Amar sem ser correspondido não é tão bom quanto amar sendo correspondido. Mas é melhor do que não amar.

O senhor acredita em universos paralelos? Por que se pensa que possa existir universos paralelos? Eles estão em outras dimensões?ou apenas distantes?

Não acho que existam Universos paralelos. Esta conjectura provém de duas fontes: relativística e quântica. São concepções distintas. Ambas advém de extensões das soluções das equações de campo das teorias, que podem ser interpretadas como representando tal realidade. Mas, para mim, as soluções das equações são indícios de possibilidades, que precisam ser averiguadas com respeito a sua realidade. E, ao que me consta, não há confirmação nenhuma da existência de Universos Paralelos. Para mim, não existem. Se existissem, não seriam afastados do nosso, pois este engloba todo o espaço existente. Eles não estariam localizados em nenhum lugar descrito pelas dimensões em que estamos imersos. Seriam outras dimensões, inexistentes em nosso Universo. Seriam completamente disjuntos do nosso. Não haveria nenhum espaço vazio entre eles. Não haveria separação entre eles, porque separação supõe a existência de alguma dimensão que fosse compartilhada por todos eles. Seria algo totalmente à parte e completamente incomunicável.

Ernesto,com relação ao sono você acha que é preciso sempre dormir na mesma hora que se costuma dormir ou não importa a hora?‎

Não sei se é preciso, mas eu não tenho hora certa de dormir e nem tempo definido de sono. Cada dia eu durmo a uma hora diferente, dependendo do que tenho para fazer. Também acordo cada dia a uma hora diferente e durmo cada dia um tempo diferente. O mesmo me acontece com o horário das refeições. Não sou nada regular. E não sinto problema nenhum com isso. Se estou com muito sono certo dia, durmo um pouco depois do almoço. Felizmente meu trabalho não tem horário definido, nem tempo definido. Em média eu trabalho de nove a dez horas por dia. Mas posso trabalhar quatro, nenhuma ou dezesseis. Tudo depende do que seja preciso fazer.

Qual sua opinião sobre Paulo Coelho?‎

Não aprecio a qualidade de sua literatura e nem tenho simpatia pela temática que ele aborda. Além do mais, acho que ele presta um desserviço à verdade por induzir as pessoas a crer em poderes acima da natureza, que não existem. Mesmo sabendo que se trata de uma ficção, o modo dele abordar dá a impressão de que seja verdade.

Ser cético então seria duvidar da verdade, nunca ter certeza de nada.‎

Essa é a modalidade radical do ceticismo, dita "pirrônica", em referência a Pirro, que não é aceita de modo geral na comunidade filosófica e que eu também rejeito. O ceticismo que abraço é o ceticismo metodológico, que não considera a verdade inacessível mas que considera difícil se saber se já se a possui. Então duvida de sua posse, exatamente para envidar o máximo de esforços para se aproximar cada vez mais dela. Claro que há questões triviais em que a posse da verdade já pode ser considerada definitiva. Mas o cético está sempre aberto a admitir um engano e fazer a correção.

Supondo que nossa consciência decorra exclusivamente da atividade do cérebro e que o avanço da tecnologia possibilite a construção de neurônios artificiais, o senhor acha plausível que no futuro possamos nos tornar "imortais" eternizando nossa vida consciente em meios artificiais não-perecíveis?

Sim, é possível. Mas ainda não temos competência tecnológica para tal. E isso ainda demora bastante. Muitas décadas ou alguns séculos.

professor, qual a sua opinião sobre o digníssimo filósofo e astrólogo olavo de carvalho?‎

Um verdadeiro desastre. Uma pessoa totalmente equivocada. Alguém que defende os mais incorretos princípios e possui as mais errôneas concepções. Além de ser totalmente deselegante no falar. Um preconceituoso e um grandessíssimo mentiroso.

O que é ser cético?

É considerar que não se tem certeza da veracidade do conhecimento, sendo pois, todo ele, passível de contestação. É admitir que todo conhecimento seja provisório e envidar constantes esforços no sentido de aprimorá-lo, para que cada vez mais se aproxime da verdade, sem nunca ter garantia total de a ter alcançado. Isso não significa supor que não se possa alcançar e sim que não se consegue saber, sem sombra de dúvida que já se tenha alcançado a explicação verdadeira. Mesmo assim, aceitar esse conhecimento provisório e trabalhar na suposição de sua veracidade, até que seja mostrado que se está equivocado. E aceitar essa contestação como algo bom para o progresso do conhecimento. Ou seja, sempre estar disposto a mudar o pensamento, cada vez que se provar que se esteja errado.

quando diz '' que o confeiteiro seja preso'' acha justo que ele seja preso por se recusar a fazer o bolo ou que é preferível que ele seja preso á fazer um bolo para um casal gay‎

 Que ele seja preso por se recusar a fazer o bolo. É um absurdo esse preconceito. Todo mundo tem o direito de se casar com quem quiser, seja de sexo for. Para mim, até, com mais de um ou uma. E tem o direito de fazer a sua festa. O confeiteiro não tem o direito de julgar as escolhas dos outros. Mesmo que ele não concorde, tem que aceitar. Do mesmo modo que eu, sendo ateu, aceito que as pessoas tenham as suas religiões. E se eu for um juiz de paz, não posso me recusar a casar quem tenha alguma religião. O que não pode ser aceito é algo que faça mal para os outros. Mas o casamento homossexual não faz mal para ninguém. Pelo contrário. faz bem aos nubentes. As convicções religiosas das pessoas não podem levar a que rejeitem convicções diferentes das suas, desde que não sejam maléficas para ninguém. Isto é, seja qual for a sua religião, você tem que aceitar as outras religiões e outras convicções não religiosas.

Se a matéria não for capaz de gerar consciência/mente, por que os adeptos do monismo fisicalista insistem em defender que o cérebro é o substrato mental e que, com a morte cerebral, perderemos também a consciência? Você toma uma posição agnóstica quanto a isso ou rejeita a ideia da "alma"?‎

A consciência não provém só da matéria. Provém da estrutura e do funcionamento dos componentes do cérebro e seus anexos. E esses componentes não são só matéria, mas também, e principalmente, campos, especialmente elétricos. O componente material e de campos, de tal modo estruturado e de tal modo funcionando, devido a sua complexidade, permitem que surja a ocorrência da mente, da qual um dos componentes é a consciência, bem como a auto-consciência, a memória, a intuição, o raciocínio, as emoções, os sentimentos, e todos os fatos da vida psíquica. Com a parada de funcionamento desse substrato, todos os seus efeitos são cessados, dentre os quais a consciência. Em outras palavras, não existe nada de "alma", como entidade sede da mente e de seus atributos.

Como manter a calma depois de uma decepção ?‎

Isso depende de uma concepção de vida já formada há muitos anos. Não há como aprender de uma hora para outra. A pessoa precisa ter um temperamento fleumático e uma personalidade estoica. Ou seja, uma pessoa que não se incomoda com as vicissitudes da vida, que tem uma auto-estima elevada. É preciso considerar que a vida, como um todo, é mais importante do que seus episódios específicos. Mas, também, tem que ser altaneiro e não se deixar levar por sentimentos de rancor, despeito ou desprezo por quem nos decepcionou. Procurar ver a ocorrência pelo lado da outra pessoa também. Estar consciente que os sentimentos negativos mais prejudicam a quem os sente do que a quem são dirigidos.

Porque você é uma pessoa tão séria? Nunca li você rindo, e em seu hangout com o Pier, você era o mais sério. Acha que toda a sua bagagem cultural o fez "sofisticado" de mais para o humor?

Não acho que eu seja tão sério assim. Sou sério e não muito sério. É o meu modo de ser, independente de meus conhecimentos. Não sei explicar a razão. Faz parte de meu temperamento e de minha personalidade, que possuem origem parcialmente genética, parcialmente ambiental. Meus pais também eram assim. Acostumei a ser assim e não acho ruim. Mas não sou casmurro. Sou jovial. Só não conto piadas e nem faço brincadeiras. Às vezes faço uma piada daquelas bem sutis e difíceis de entender, que a maioria não acha graça. É só para nerds. Aprecio o humor, desde que não seja escrachado e, muito menos, chulo. Humor debochado e preconceituoso em abomino mesmo e não acho graça nenhuma. Não consigo ser diferente e nem tento, porque não quero ser diferente.

Concorda com essa resposta na parte "humeniano da causa"? http://ask.fm/GuiTomishiyo/answer/105201654612‎

O sentido humeniano é o de que causa é algo que sempre está presente precedendo algum evento. No caso da emissão de fótons, é preciso que o átomo esteja excitado. Então a excitação é um evento que sempre precede a emissão. Mas ela não a determina. Um átomo pode ficar indefinidamente excitado e nunca emitir. Por isso a excitação, no sentido estrito, não é uma causa e sim uma condição. Ou seja, ela é necessária, mas não é suficiente. Não há nada que baste para que um átomo emita fótons. Por isso diz-se que a emissão é um evento fortuito, isto é, incausaudo. Considerar a excitação como causa da emissão é cometer o mesmo erro que dizer que "para morrer, basta estar vivo". Claro que não basta, senão todo vivo logo morreria. Mas é preciso se estar vivo para morrer. Logo, não se pode dizer que o nascimento é a causa da morte.

Acha necessário, pelo menos no estado em que a sociedade atual se encontra, declarar-se ateu? Quando eu conheço alguém, eu não me apresento, por exemplo, como um não-auxiliar de cozinha.Ou seja, um não-praticante ativo(?). Levantei essa questão, mesmo sendo ateu, e gostaria de saber o que acha.‎

Não é a mesma situação. Claro que se não se inquirir a respeito, não ha necessidade de se dizer a condição religiosa que se tem, mesmo porque isso é irrelevante para a maioria dos propósitos. Mas se o tema vier à baila, sim, deve-se dizer que concepção religiosa se tem. Por outro lado, como o ateísmo é uma condição ainda incomum, acho que vale a pena mencioná-la em apresentações, como, por exemplo, em meu perfil neste Ask. Isso funciona como uma propaganda, já que, para mim, penso que todos devam se tornar ateus, pois considero essa concepção a mais verdadeira. Portanto, mesmo não sendo necessário, penso que seja bom, para difundir o ateísmo.

Professor, se o acaso possibilita tudo, então ele tem o poder, o poder de possibilitar tudo?‎

Não. Possibilitar não é ter poder. O verbo poder tem três significados. O primeiro é possibilitar, isto é, ter condição de se realizar algo. O segundo é ter capacidade de fazer e o terceiro é ter permissão de fazer. Quando digo que posso fazer algo posso estar dizendo que esta possibilidade existe, mas eu não seja capaz, ou que eu seja capaz, mas não tenha permissão, ou que eu tenha permissão, mas não isso não seja possível para mim, ou eu não tenha capacidade. No caso em tela, a palavra poder está sendo usada no sentido de ser capaz. Pois bem, não é o acaso que é capaz de fazer nada. O acaso é o que possibilita que tudo seja feito, mas para que seja feito mesmo, é preciso que ocorram condições propícias. Quanto à permissão, isso não acontece nos fenômenos naturais, já que, para a natureza, tudo que seja possível e que se tenha capacidade de ser realizado é permitido. Não há proibições na natureza, isto é, proibições, digamos, morais. Há proibições no sentido de não haver possibilidade, como, por exemplo, fazer um moto contínuo. Poder, como o que se diz a respeito de Deus, é ter capacidade. E isso exige decisão, o que o acaso não faz. Acaso não é um agente de nada. É apenas a palavra que descreve o fato de algo acontecer sem que tenha sido provocado, isto é, fortuitamente. Tem um tipo de semântica análoga à da palavra "nada", que não descreve coisa alguma.

O acaso é fruto de uma pequena probabilidade, quase nula, quase impossível, mas é no pouco que se pode esperar dele que os ateus se fiam. Esse é o problema. A probabilidade de qualquer vida ocorrer no universo, ainda mais inteligente, é absurda. Você não entende o quão ínfima é a probabilidade. Nula‎

Uma probabilidade extremamente pequena não é nula. E não é preciso esperar um tempo imenso para que se dê a ocorrência. Veja o caso das loterias. A probabilidade de ganhar é muito pequena, por exemplo, sozinho na Mega Sena. Mas há muitos que ganham. E eles tinham a mesma probabilidade de ganhar do que os que perderam. Assim, mesue seja muito pouco provável que, passo por passo, a vida surja da matéria inanimada, não é impossível e é perfeitamente razoável considerar que foi o que ocorreu. Além disso é preciso entender que não é a probabilidade de todos os átomos do nosso corpo, por acaso, se reúnam nas posições corretas para nos construir que tem que ser calculada. É a probabilidade de que alguns átomos formem aminoácidos, e bases nucleotídicas. Depois que aminoácidos e bases, já formadas, se reúnam e formem proteínas e ácidos nucléicos e assim por diante, nível por nível, o resultado de cada um sendo o ponto de partida do próximo, ao longo de centenas de milhões de anos. Isso aumenta extremamente a probabilidade. Veja esse cálculo:
http://www.ruckert.pro.br/blog/?p=819mo q

Ernesto,eu estou estudando física porque me interessa muito e eu gostaria de entrar na UFMG.Em toda a sua experiência que você teve e ainda continua tendo,como que era o seu ritmo de estudos?

Cada pessoa tem o seu modo de aprender. Eu me concentrava muito na própria aula. Participava ativamente, perguntando sempre e raciocinando com o professor. Para tal eu me inteirava da matéria antes da aula e anotava as dúvidas. Acho isso essencial. Entrar para uma aula sem saber o que vai ser dado é um desastre. Depois eu estudava cerca de meia hora para cada aula dada, Isso me consumia apenas umas duas ou três horas por dia fora da aula, em média. Bem pouco. Aí, eu fazia resumos, como se fosse ensinar aquilo. É a melhor forma de aprender. Preparar aulas do que está estudando. Isso eu recomendo fortemente mesmo. Pense como um professor. Que você tem que saber aquilo sem escapatória para ensinar. E não se baseie nunca em um livro só. Gaste dinheiro mas compre vários de cada matéria. E nunca os jogue fora. Conserve-os para o resto da vida.

Ernesto, qual o nível de matemática é necessária para entender a coleção: Lições de física - Feynman ? Cálculo Diferencial e Integral é suficiente ? Grato.‎

Não. Além do cálculo diferencial e integral tem-se que saber cálculo vetorial e, certamente, muita geometria e trigonometria. Mas parte da matemática mais avançada necessária ele mesmo ensina no livro.

O deus dos ateus é: o acaso. O acaso é fruto de uma pequena probabilidade, quase nula, quase impossível, mas é no pouco que se pode esperar dele que os ateus se fiam; depositam sua fé nele tal qual teístas fazem com o seus Deuses. No final das contas, ateus e teístas não são tão diferentes.‎

De modo nenhum. O acaso não é um deus porque o acaso não tem poder. Ele não determina nada. Ele só possibilita tudo. E o que acontece não se pode prever. Pode ser qualquer coisa. O que já aconteceu é porque coincidiu de acontecer e poderia não ter acontecido. Não há razão nenhuma para que tenha acontecido. Não há vontade nenhuma que resolva o que vai ocorrer. Não há inteligência nenhuma que planeje o que vai acontecer. A diferença em relação à concepção de um Deus é extremamente grande.

Professor,ouvi em algum lugar que a velocidade necessária para entrar na orbita da terra é de 26000km/h. Se verdade,por que tem que ser essa velocidade? O que aconteceria se a velocidade fosse menor(15000-20000km/h)?‎

A velocidade orbital depende da altura da órbita. Basta igualar a aceleração centrípeta com o campo gravitacional, isto é v²/r = GM/r², em que r é o raio da órbita (raio da Terra mais a altura), v é a velocidade, M é a massa da Terra e G a constante da gravitação universal. Se a velocidade no ponto em que o satélite for colocado em órbita for menor do que a necessária ele descreverá uma órbita elíptica em que aquele ponto será o apogeu (se a velocidade for horizontal). Se for maior, uma órbita elíptica em que aquele ponto será o perigeu. Pode ser que essas órbitas o façam penetrar na atmosfera, o que causará o seu freiamento e sua queda ao solo. A velocidade de 26.000 km/h = 7.222 m/s corresponde a uma órbita de 1.330 km de altura, com um período orbital de uma hora e 52 minutos.

Carlison Raffael: doutora ta espirada hoje, mais concordo plenamente. - Esse é um comentário de um estudante de medicina. Ele faz o curso na Bolívia, onde não há qualquer processo seletivo, basta pagar. O que acha de uma pessoa que mal sabe escrever ser um futuro médico? O que acha desses cursos?‎

Não tenho informações sobre a qualidade dos cursos de medicina na Bolívia. Claro que não se pode aceitar um médico analfabeto. Se é assim lá, não se pode aceitar que médicos de lá exerçam a medicina aqui, sem uma revalidação. Se bem que aqui também tem faculdades em que a seleção é só para constar e a aprovação depende só dos pagamentos. O que seria preciso é que o exercício da medicina fosse controlado por um exame do tipo que a OAB aplica aos bacharéis em direito para que possam exercer a advocacia.

"A razão de ter algo em vez de nada não existe. Não há razão para isso. É só por acaso." Por isso as pessoas acreditam em Deus.. olhar para a imensidão do universo, para a existência e para a ordem complexa da vida, achando que é tudo fruto de acaso beira o absurdo. Por isso Deus é muito mais prov‎

Não é absurdo nenhum. O acaso é capaz de qualquer proeza. Supor a interveniência de um Deus é que é uma consideração muito mais incompreensível e absurda. A formação de tudo por meios inteiramente naturas e fortuitos é muito mais plausível do que a suposição de alguma intervenção inteligente. A concepção de uma entidade com os atributos de Deus, sendo extrínseca ao Universo, mas capaz de intervir sobre ele é algo extremamente implausível e inverossímil. Se não se tem explicação de como se deu a ocorrência do surgimento do conteúdo do Universo, também não se tem de como esse conteúdo teria sido produzido pela vontade de algum Deus. Acho muito mais aceitável considerar que surgiu ao acaso, sem ter sido provocado por nada e sem provir da nada, como, aliás, também teria sido se tivesse sido causado por uma ação divina. O absurdo é supor a existência de um Deus e não um surgimento ao acaso.

Como foi que Isaac Newton descobriu a lei da gravidade?‎

Comparando a aceleração de queda dos corpos na superfície da Terra com a aceleração centrípeta da Lua em seu movimento em torno dela. O que ele considerou é que essa aceleração centrípeta era devida à força de gravidade da Terra sobre a Lua e que essa força sobre um corpo na superfície da Terra seria dada como se toda a massa da Terra se concentrasse em seu centro. Para provar que isso é o que acontecia ele teve que somar todas as contribuições das atrações de cada pedaço da Terra sobre um corpo em sua superfície. Para fazer isso ele teve que inventar a matemática necessária, que é o cálculo integral. Comparando os valores ele pode concluir que a força variava com o inverso do quadrado da distância. Por outro lado, comparando a aceleração centrípeta do movimento dos diversos planetas em redor do Sol. com a distância que eles estavam dele, pode tirar a mesma conclusão. A proporcionalidade com o produto das massas advém do princípio de superposição, que pode ser facilmente verificado pela constatação que um corpo de massa dupla tem peso duplo, bem como pela lei da ação e da reação. Juntando tudo só faltava achar a constante de proporcionalidade, que Newton não sabia, mas que foi achada em 1873 a partir da experiência de Cavendish, realizada em 1798, 71 anos depois da morte de Newton. Além disso, Newton mostrou que sua lei da gravitação implicava em que as órbitas dos planetas seriam elípticas, como o tinha mostrado Kepler, a partir das medidas astronômicas de Tycho Brahe.

Oi! Se a luz tem velocidade constante independente do referencial, por que ela desacelera em meios um pouco densos. Ou seja, por que a lua tem uma velocidade c no espaço, mas uma um pouco menor na atmosfera terrestre.‎

A velocidade da luz continua sendo sempre a mesma. O que acontece é que, dentro de um meio material, a luz é absorvida e reemitida pelos átomos enquanto caminha e isso provoca um atraso em seu tempo de trajeto de um ponto a outro. Então se calcula uma velocidade líquida, incluindo esses atrasos, que se torna menor do que a velocidade no vácuo. Mas entre um átomo e outro, o que existe é vácuo, logo a luz passa de um ao outro com sua velocidade de vácuo.

Prof,és a favor do ensino religioso nos colégios?‎

Religioso não, mas de religiões sim. Sob um aspecto antropológico, sociológico, histórico e filosófico. Todas as principais. Numa disciplina que apresentasse suas doutrinas, suas práticas, suas concepções, sua ética, as vivências de seus seguidores, suas qualidades e seus defeitos. Estudados de maneira completamente isenta e livre. Incluindo, também, as concepções ateístas e agnósticas. Isso seria muito importante para que os jovens pudesse se decidir a respeito de qual delas aderir, ou a nenhuma, com pleno conhecimento e não porque a família ou o grupo social seja filiado a esta ou aquela.

Professor,se a é preciso a luz para ver os objetos,por que objetos dentro de sombras ainda podem ser visto? Por que as sombra não não totalmente escuras? Se algum corpo impede a passagem da luz.‎

Só são vistos objetos dentro de sombras que não sejam totalmente escuras. Nelas há incidência de luz, só que com menor intensidade. Quando não há luz nenhuma, não se vê nada mesmo. A luz dessas sombras é a luz difusa, proveniente de outras fontes. É o que acontece com a Lua Nova, em que se vê uma claridade tênue na parte da Lua não iluminada pelo Sol. Trata-se da luz que a Terra manda para a Lua ao refletir a luz do Sol.

"mesmo as pessoas que consideram que o Universo tenha sido criado por um Deus, consideram que ele o fez sem ter de que" Isto porque foi Deus que causou o universo e trouxe o próprio "ter de que" a existência. Sem causa primeira, a própria realidade física não existiria. Por que teria algo dif. nada?‎

No momento em que Deus fez surgir o conteúdo do Universo, ele não o trouxe de nada pré-existente, pois não havia. Isso mostra que, qualquer que seja a consideração causal para essa ocorrência, ela se deu assim, ou seja com o aparecimento de tudo sem provir de nada. A diferença é se houve um agente causador disso ou não. O importante é saber que causa não é uma necessidade. Pode ter havido esse surgimento sem causa. Essa é que é a grande dificuldade dos criacionistas. Eles não admitem eventos que não sejam efeitos de causas. Isso não é, de modo nenhum, uma exigência científica. A ciência não considera que todo evento seja um efeito. A existência de eventos fortuitos, que não sejam efeitos, é sobejamente conhecida e não tem problema nenhum. Isso não quer dizer que nenhum evento não tenha causa, mas, apenas, que não é preciso que tenham causa. A razão de ter algo em vez de nada não existe. Não há razão para isso. É só por acaso.

Professor é correto usar o termo '' FORÇA '' Para se referir a gravidade? Posso estudar e falar para alguém tranquilamente sobre uma suposta '' força '' da gravidade?? que outro nome posso usar para substituir o termo '' Força gravitacional "? q eu sinto estar errado. obrigado‎

Rigorosamente não é correto, pois, de fato, a gravidade não é uma interação e força é a medida de intensidade de uma interação. O correto é considerar a ação da gravidade como um fenômeno inercial ocorrendo em um espaço-tempo com curvatura. A curvatura é fornecida pelo total de massa-energia do conteúdo do espaço e o movimento inercial se dá nas geodésicas desse espaço-tempo encurvado. Só que esse tratamento é matematicamente muito mais complicado, de modo que, para efeitos práticos, desde que a curvatura não seja grande, pode-se fazer uso do modelo de espaço-plano, no qual a gravidade seja considerada uma interação. Isso é o que se usa, até mesmo, para o cálculo de órbitas de foguetes e satélites. Os casos em que a curvatura do espaço-tempo precisa ser levada em consideração se dão nos Buracos Negros e em dispositivos muito sensíveis, como é o caso dos sistemas de GPS.

Deus não poderia criar o big bang por quê antes do big bang não existiria o tempo?

Deus, se existisse, seria uma entidade extrínseca ao Universo. Logo não estaria imerso no espaço e nem no tempo. Não teria volume nem duração. Também não teria massa, forma, aspecto, energia e nenhum atributo físico de sistemas. Portanto, se houve um jeito dele fazer surgir o Universo, e com ele o tempo e o espaço, a inexistência de tempo prévio não seria problema. A questão difícil é conceber como se poderia fazer uma interveniência de algo extrínseco ao Universo nele,mesmo que para dar o seu surgimento. Aliás, a própria concepção da existência de algo que não pertença ao Universo é problemática. Ainda mais essa entidade estabelecer contato com o Universo. Acho que a concepção da existência de Deus é imensamente mais problemática do que a consideração de que o Universo tenha surgido sem causa e sem ter do que provir.

Professor, o que quer dizer alfabeto? Estava conversando com minha mãe e ela acha que o certo é alfabeta "começo e fim". Enfim, o que você me diz?‎

Alfabeto é a coleção de signos que são usados para representar pictoricamente a fala. Esses signos, chamados glifos, podem ser fonéticos, isto é, representarem sons, que são os fonemas, ou ideográficos, isto é representarem idéias, que são os ideogramas. Esse nome vem das duas primeiras letras da escrita grega, que são alfa e beta (a última é omega). Como a palavra que representa esse conjunto foi considerada masculina, ao ser latinizada, passou a se escrever alfabeto em vez de alfabeta. Os alfabetos do Egito antigo, da China, do Japão e da Coréia são ideográficos.

Acreditar que o universo surgiu a partir de nada incausado é pior que mágica. Quando um mágico tira um coelho da cartola, pelo menos há um mágico. É realmente nisso que o senhor acredita?‎

Sim. É nisso. E isso é perfeitamente possível e não é mágica. É um surgimento de algo sem que tenha do que provir. Aliás, mesmo as pessoas que consideram que o Universo tenha sido criado por um Deus, consideram que ele o fez sem ter de que. Ou seja, a criação é apenas a introdução de um fator causador do surgimento e não de algo a partir de que o surgimento se deu. Inclusive porque, se houvesse algo que teria se transformado em Universo, isso já seria o Universo e a criação não seria um surgimento e sim uma transformação. Para se considerar que antes não havia Universo e, em dado momento, passou a haver, tem-se que considerar que surgiu de nada, quer de modo fortuito, quer de modo provocado. Mas, se foi provocado, teria que ser por um agente extrínseco ao Universo, uma vez que ele não existia. Não vejo problema em considerar um surgimento sem ter do que provir. Isso, inclusive, não contraria nenhuma lei de conservação, pois essas leis descreve o comportamento do universo existente. Não existindo, também não existem leis, logo tudo é permitido.

A espação do big bang foi de fato mais rápido do que a velocidade da luz, concorda? Ou seja o nada é mais rápido do que a matéria?

Sim, mas não é uma expansão de nada. É o espaço que se expandiu. E espaço não é nada. É alguma coisa. Nada é a ausência de tudo. Até de espaço vazio. Espaço não tem massa e nem energia. Mas tem extensão. Então é algo que possui propriedades. Tem também curvatura (ou não). Tem topologia. Pode ter torção e cisalhamento. Mesmo que seja vazio. Mas não existe espaço vazio no Universo. Todo espaço existente é preenchido, pelo menos por campo e radiação, mesmo que não contenha matéria. Espaço sem matéria chama-se vácuo. Mas vácuo não é vazio. Contém campos e radiação, que possuem energia e várias outras propriedades. A separação entre dois pontos do espaço pode variar sem que eles se movam com uma taxa de variação superior à velocidade da luz. Só que, como não se trata da variação da posição de algo em relação a algum referencial, mas da própria escala do referencial, não se trata de uma velocidade. Então não está sujeita às restrições da relatividade especial a respeito do limite de velocidade.

Sim, eu experimento ouvir de todo tipo de música. Mas só volto a ouvir se achar agradável a meus ouvidos, isto é se eu gostar. Isso acontece tanto com música popular quanto com música clássica. Às vezes eu ouço só para ter conhecimento da existência e de como é, por curiosidade, mesmo que não me traga prazer. O prazer que eu fruo na audição de uma música não é só o prazer sensorial da beleza melódica, harmônica, rítmica, tímbrica ou da beleza poética da letra. É também um prazer intelectual haurido da análise estrutural da música, de sua construção e, se for o caso, da mensagem contida no conteúdo da letra e de sua proposta em termos de impacto a provocar. Isso vale também para a apreciação de uma obra de arte plástica, de uma dança, de uma peça de teatro, um filme ou qualquer outra obra de arte.

Atualmente eu quase não estudo mais física, pois já estudei muito ao longo da vida. Só quando me aparece um tema que ainda não tenho conhecimento é que eu vou estudá-lo, para entendê-lo. Já estou numa situação que, se me fecharem em um quarto com água, alimento, papel e lápis, sou capaz de escrever, de cabeça, uma coleção de quatro volumes de física geral, tipo Halliday, sem consultar nada. Ou mesmo um livro de mecânica clássica, termodinâmica, eletromagnetismo, ótica, física quântica ou relatividade geral. Nesses últimos é possível que um ponto ou outro eu precise conferir algo, consultando. Mas tenho todo o esquema na cabeça do que precisa constar no livro e como desenvolvê-lo. Física do Ensino Médio, então, sem o menor problema. Tenho até muitas idéias de abordagens diferentes que usei ao longo de meus anos de magistério que facilitam o entendimento. Gostaria muito de poder me dedicar a isso, se me pagassem para ficar por conta de escrever, pois preciso do dinheiro que ganho com meu trabalho.

Ernesto, você costuma explorar todo tipo de variações de estilos musicais? Digo, você tenta entender sua concepção ou conceito artístico ou normalmente prefere ficar na sua "zona de conforto musical"? (não interprete com tom de critica essa pergunta, é uma dúvida legítima)‎

Sim, eu experimento ouvir de todo tipo de música. Mas só volto a ouvir se achar agradável a meus ouvidos, isto é se eu gostar. Isso acontece tanto com música popular quanto com música clássica. Às vezes eu ouço só para ter conhecimento da existência e de como é, por curiosidade, mesmo que não me traga prazer. O prazer que eu fruo na audição de uma música não é só o prazer sensorial da beleza melódica, harmônica, rítmica, tímbrica ou da beleza poética da letra. É também um prazer intelectual haurido da análise estrutural da música, de sua construção e, se for o caso, da mensagem contida no conteúdo da letra e de sua proposta em termos de impacto a provocar. Isso vale também para a apreciação de uma obra de arte plástica, de uma dança, de uma peça de teatro, um filme ou qualquer outra obra de arte.

Quando você diz que tudo o que existe é tudo o que pode ser percebido você não está usando mão de um argumento sofista?

De modo nenhum. Se algo existe, então é possível, direta ou indiretamente, detectar sua existência. Se algo não é passível de ser detectado, mesmo que por seus efeitos indiretos, não se pode afirmar que exista, mas apenas supor. Por exemplo, os quarks, por muito tempo foram considerados uma hipótese, até que experimentos os puderam detectar, indiretamente. O mesmo acontece, atualmente com as supercordas. Ainda não se pode dizer que existam. Deuses e espíritos estão na mesma situação. São hipóteses não confirmadas. Enquanto não houver confirmação não se pode dizer que existam. Todavia temos duas categorias de realidades. As objetivas, independentes de um sujeito que as considere em sua mente e as puramente mentais ou abstratas, como os conceitos. O conceito de Deus existe, mas a entidade objetiva não se sabe. Abstrações têm a sua realidade própria, mas não existem sem que haja mentes que as concebam. É o caso dos números, dos valores, das normas. Mas as abstrações podem ser percebidas mentalmente, o que confirma sua existência como abstração e não como entidade objetiva do mundo exterior. Deus e espíritos, se existirem, seriam entidades objetivas, fora das mentes. Isso não é garantido. Assim considerando, esse critério de realidade (isto é, do que existe) não é, de forma nenhuma, um sofisma.

Professor, o quê acha dos Europeus e descendentes que temem que o gene da "raça" branca seja "extinto"? A maioria são contra imigrantes, a mistura de raças, etc. Dizem que querem "preservar" a cultura branca‎

Isso é bobice. A cultura de qualquer povo, de que etnia seja, pode ser preservada sem a necessidade de se manter uma pureza racial e nem de se segregar essa cultura do contato com outras culturas. É bom que o mundo seja cada vez mais globalizado e miscigenado racialmente. Cada cultura tem seus valores que podem beneficiar outras culturas. O que seja característico de cada uma pode ser preservado como folclore e cultivado em ocasiões e lugares especiais. A tendência mundial é haver, realmente, uma rarefação da dita "pureza racial", pois, cada vez mais, o isolamento que produziu essas diferenciações, ao longo de dezenas de milhares de anos na pré-história, deixa de existir. Mas isso é que é bom. Que a humanidade toda seja uma só em suas características. Isso deverá levar a um ser humano pardo e de olhos meio amendoados, com cabelos ondulados e castanhos, bem como olhos castanhos. Pessoas louras, de olhos azuis, bem como negras, de cabelo crespo, ou de feições completamente orientais, pelo que percebo, não existirão mais dentro de poucas dezenas de milhares de anos. E, possivelmente, o mundo inteiro falará um único idioma e será uma única nação. Totalmente em paz e harmonia, próspero e feliz, sem pobreza, sem doença e sem ignorância nenhuma. E, ainda acho, sem religiões, sem governos, sem dinheiro e sem propriedade. Mas sofisticadamente culto e extremamente avançado tecnologicamente. E finalmente, com um altíssimo comportamento ético, de modo que não existam crimes de espécia alguma.

Professor,em queda livre a aceleração é constante?Pois no ensino médio os problemas envolvendo queda livre só se usa a aceleração da gravidade. É realmente certo só considerar a aceleração da gravidade?‎

Não é constante. A aceleração devida à gravidade varia com a distância ao centro da Terra. Para uma queda de baixa altura a variação é desprezível, mas não se a queda se der de vários quilômetros de altura. Além do mais, é quase impossível, próximo da Terra, ocorrer uma "queda livre", uma vez que a atmosfera aplica duas forças ao móvel em queda, o empuxo e a resistência do ar. A queda só será livre fora da atmosfera. O empuxo depende da densidade do ar, que varia com a altitude e com a temperatura e a resistência do ar depende da velocidade, da forma do objeto em queda e da textura de sua superfície. Isso tudo faz com que o estudo teórico do movimento de queda de corpos seja bem complicado, requerendo a solução de uma equação diferencial. Além do mais há que se considerar que, em relação ao referencial do solo da Terra, que não é inercial, a segunda lei de Newton não se aplica rigorosamente, sendo preciso incluir dois fatores para calcular a aceleração efetiva da gravidade, a força centrífuga e a força de Coriolis, advindas da rotação da Terra. É a força de Coriolis que provoca a rotação dos ventos em um furacão. O estudo completo desse problema é interessantíssimo, mas não costuma ser estudado nem na Física Geral dos cursos de Engenharia e sim na Mecânica Clássica da parte profissionalizante do Curso de Física.

Sabe-se que a o motivo da Lua, para a Mecânica Clássica, (o mesmo se aplica à outros astros) cair na superfície da Terra é porque ela tem uma velocidade (v) tangencial à Resultante Centrípeta (que é de origem gravítica). Mas de onde veio essa velocidade inicial (v) da Lua, que não a deixa "cair aqui‎

No caso particular da Lua da Terra, ela deve ter sido formada devido a uma colisão da Terra com outro planeta, e, portanto, seu material já tinha essa velocidade antes de ser agregado para formar a Lua. Em geral, na formação de planetas, Seu material provém de gás acrescido do espaço e, ao se unir, isso não acontece de modo completamente radial, fazendo com que o material que dará origem à estrela adquira uma rotação. À medida que o material se condensa, as partículas de maior velocidade são ejetadas, formando um disco, semelhante aos anéis de Saturno. Nesse disco ocorrem, por sua vez, aglomerações, pela gravidade recíproca, que formarão os planetas, da mesma forma, em rotação. Dos planetas, em geral, surgem discos nos quais se formam suas luas. A da Terra é uma exceção, devido a seu grande tamanho em relação ao planeta. A velocidade tangencial das luas e dos planetas, portanto, é algo que o material de que foram feitos já possuía, antes de se agregar.

Professor,Se eu quiser calcular a força que um corpo atinge um solo(sendo que ele estava em queda livre) usando f=ma,qual será a aceleração? Como calcular essa aceleração? Estou confuso pois não sei se uso a aceleração da gravidade ou tenho que calcular a aceleração e somar com g

Para calcular a aceleração você tem que saber o quanto o corpo ainda avança para baixo depois de tocar o chão, ou o tempo que ele leva para parar depois que toca o chão. Isso não é fácil de achar. Tem que fazer uma filmagem de alta frequência e depois passar em câmara lenta. A partir dessas informações ou você acha a aceleração pela variação da velocidade dividida pelo tempo ou usa a relação vdv = ads. Como essa aceleração de freiamento é muito maior do que a da gravidade, pode desprezar a gravidade no cálculo. Claro que, para começar, você tem que achar a velocidade que ele atinge imediatamente antes de tocar o solo em sua queda. Se não houver resistência do ar isso é fácil, com as equações de movimento uniformemente acelerado do Ensino Médio. Se houver resistência do ar complica, pois essa é oposta ao movimento e proporcional ao quadrado da velocidade para grandes velocidades e à velocidade para pequenas velocidades. Então se tem que resolver uma equação diferencial, que depende de conhecimento de cálculo infinitesimal.

Professor, o que acha da Social Democracia? Ela serviria como uma transição para a anarquia?

Acho ótima, por enquanto. E, realmente, a melhor opção para se atingir a anarquia. Mas tem que ser acompanhada de uma pulverização do capitalismo, com a gradativa abolição do trabalho assalariado e a conversão de todo trabalhador em sócio do empreendimento em que trabalha. Assim todos serão patrões. Bem como uma firme transformação dos empreendimentos em empreendimentos cooperativos e não empresariais.

Ernesto, para você - um ateu de uma convicção forte - o que existe?‎

Tudo o que se pode perceber pelos sentidos e por meio de instrumentos existe. Isso é, toda essa realidade fora de nossas mentes. Campos, matéria, radiação, espaço, tempo, estruturas, ocorrências. Bem como abstrações. Só não existem entidades sobrenaturais.

Professor,entre física,matemática e química,qual está mais avançada? Será possível algum dia elas estarem 100% completas. Como seria esse mundo?‎

Todas estão igualmente avançadas, mas nunca estarão completas. Nem se sabe que fração do conhecimento a respeito delas já é dominado, mas, para mim, não chega nem a 10%. É preciso considerar que a humanidade ainda pode existir neste planeta por mais de uma dezena de milhão de anos. Nem dá para imaginar o mundo daqui a dez milhões de anos.

Por que há algo em vez de nada?‎

Não há razão nenhuma. Simplesmente poderia não haver nada. O surgimento de tudo se deu por acaso, sem razão, sem propósito, sem projeto, sem causa e sem ter do que provir. Como eu posso garantir isso? Não posso! Mas é o que eu considero, uma vez que não há nada que aponte nenhuma razão nem propósito. Uma boa discussão das variadas propostas de explicação disso pode ser achada no livro "Por que o Mundo Existe?" de Jim Holt, que li recentemente (Intrínseca). Mas ele não acha a resposta.

Samsara é o ciclo renascimento, não é reencarnação. Para reencarnar, é necessário ter um eu, uma das bases do Budismo é o conceito de Anatta (não-eu, inexistência do "eu"). E não é a supressão do desejo em si, mas o sentimento de saber a inutilidade de muitos deles e quais são hábeis ou inábeis. :P

Que seja. Mesmo assim, não aceito renascimento nenhum. E nem esse conceito de "não-eu". O que nós somos está em nosso "eu" e ele é único, formado em nossa gestação e extinto com nossa morte. Não há renascimento nenhum. Também discordo da inutilidade do desejo. Acho que desejo é uma coisa boa. O que não é bom é se ter uma sofreguidão por sua realização que cause uma grande frustração quando não realizado. Há que se ter serenidade no desejo e um comportamento estoico nas vicissitudes. Isso eu apoio. Mas o que eu penso é que todas essas concepções de vida virtuosa não precisam ser caracterizadas como religião nenhuma, mas apenas como "filosofia de vida". Para mim isso é muito mais valioso do que a adesão a qualquer religião, especialmente porque se trata de concepções abertas a revisões e acatadas em um processo de livre pensamento e reflexão. Como aconteceu com o próprio Buda. O que eu acho é que cada um deva ser um Buda e não seguir o que Buda pregou. Pode-se, é claro, conhecer tudo o que pregaram Buda, Lao-Tzé, Confúcio, Salomão, Zaratustra, Jesus, Maomé, Lutero e quais líderes e profetas sejam. Mas cada um tem que ser o profeta de si mesmo e achar o seu próprio caminho de vida, sua cosmovisão.

Sei que é clichê e importuno, mas como o senhor responderia a Teoria do Design Inteligente, que diz que certas características biológicas desafiam o padrão darwiniano de “coincidências fortuitas”, pois aparentam desenhadas, sendo só uma causa inteligente explicação para sua complexidade estrutural?

Claro que não. Não há razão nenhuma para que qualquer estrutura biológica, por mais complexa que seja, exija um projeto inteligente para se estabelecer. Qualquer estrutura pode ser formada por acaso. O acaso é capaz de qualquer coisa, mesmo que seja extremamente improvável, desde que não seja impossível. Além do mais, na evolução, as estruturas não se formam de modo acabado de uma vez. Elas vão evoluindo, de espécie para espécie, a cada etapa com um acréscimo de complexidade, até que se apresentem como são atualmente. E ainda evoluirão mais ainda, para o futuro.

O que pensa da frase do teólogo Leonardo Boff que diz que quando a Ciência provar a existência de Deus ele se torna ateu?‎

Acho muito incoerente da parte dele. Pelo contrário, se for provado que Deus existe, eu, que sou ateu, deixarei de sê-lo. Pois eu sou ateu, justamente porque não há evidências, provas e nem indícios de que Deus exista. E eu não acho que nada deva ser admitido apenas por fé, pois fé é algo que não tem o menor cabimento. Mas, sendo demonstrado inequivocamente, é claro que eu aceitarei.

Professor, o que acha do Budismo?‎

Muito interessante no aspecto de conduta e relacionamento humano. Mas completamente equivocado nos conceitos de sansara (reencarnação) e karma. Por outro lado, também não concordo que a supressão do desejo seja a solução para o problema do sofrimento. O que resolve não é podar o desejo, mas a frustração por não o ter atingido.

No seu ponto de vista, como você diferencia protestantes históricos como Luteranos, Anglicanos, Batistas, Presbiterianos e Metodistas dos pentecostais e neo-pentecostais como Assembléia, Quadrangular, Universal, Mundial, etc?

A diferença principal está na concepção pentecostal, isto é, eles consideram que são capazes de agir pela força do Espírito Santo e fazer milagres. Veja isto:
http://pt.wikipedia.org/wiki/Pentecostalismo

Quais foram as tentativas, dentro da ciência, de provar que deus existe? Até onde se chegou?

Não existem tentativas científicas de provar a existência de Deus e sim filosóficas. As principais são as cinco vias de Tomás de Aquino. Atualmente existem outras provas sendo propostas, como as de Gödel e de Plantinga. Mas são falaciosas. Pesquise na internet. Já refutei algumas em meus blogs e há sites em que outras refutações são apresentadas.

O que Einstein quis dizer é que imaginar Deus criando o mundo como dita a religião é ignorância, enquanto que explicar o mecanismo do mundo sem agente, Deus, é conceber um mundo amputado. Afinal, olhe para o esplendor do universo... Desculpe por não ter conseguido encaixar o texto anterior no espaço‎

O Universo tanto é esplendoroso quanto é horripilante. O tanto que há de beleza, há de feiura. O tanto que há de bondade, há de crueldade. Logo não é um testemunho de uma criação por um ser perfeito. As imperfeições do Universo são inúmeras. Se ele tivesse sido obra de um criador, este seria um incompetente. Mas, uma vez que se formou por acaso, é de se esperar mesmo que exiba uma série de defeitos de construção. Por exemplo, as doenças, só para citar o que nos toca diretamente.

Qual a diferença entre livros de consulta e de leitura?‎

Livro de leitura é aquele que você começa na primeira página e vai até a última, como um romance ou um livro de divulgação científica. Livro de consulta é aquele que você vai a páginas específicas onde se encontra um assunto que você quer saber de cada vez, não precisando ler o livro todo, como dicionários, enciclopédias, manuais e livros didáticos.

"O agnóstico pode achar que deus existe ou que não existe. O que o diferencia é que ele acha que isso não é possível de se saber." Professor, não seria, então, certo dizer que todos são agnósticos ? Pois, não há como provar nenhum dos dois lados da moeda. Correto

Não. O agnóstico considera que essa prova não seja possível nunca. O cético, como eu, vê que essa prova não existe, dentro dos atuais conhecimentos, mas não que seja impossível de ser obtida. Os gnósticos, por sua vez, consideram que já está provado, ou que existe ou que não existe.

O que você acha da teoria das cordas?‎

Muito interessante mas, em minha opinião, implausível. Não tenho argumentos para contestá-la e nem para defendê-la. Comprei dois livros a respeito. Teóricos mesmos, em nível de pós-doutorado. Mas ainda não me dispus a estudá-la a fundo, já que é bem difícil mesmo. Meu sentimento é de que ela não seja correspondente à realidade e que a gravitação não seja uma interação. Isto é, que a dita "Unificação das Interações" não existe. Já li livros de divulgação a respeito, como os do Michio Kaku e do Brian Greene. São muito bem escritos, mas não me convenceram de que seja aquilo mesmo

concorda que um agnóstico é um ateu sem coragem ou certeza?

Não. O agnóstico pode achar que deus existe ou que não existe. O que o diferencia é que ele acha que isso não é possível de se saber. Por outro lado o gnóstico sabe ou que deus existe ou que não existe. E o cético acha que se pode saber, mas duvida desse conhecimento. Tanto o agnóstico, quanto o cético e o gnóstico podem ser ateus ou crentes. Eu sou um ateu cético. Pode haver também ateu agnóstico, ateu gnóstico, crente agnóstico, crente gnóstico e crente cético.

Entropia é uma grandeza que mede exatamente o que? ela só tem magnitude ou também pode ser representada por vetores?‎

Entropia é uma grandeza escalar cuja variação em um sistema é dada, termodinamicamente, pela razão do calor que ele cede ou recebe pela temperatura absoluta em que esse calor é cedido ou recebido, em um processo reversível. Como se trata de uma grandeza de estado, pode-se escolher qualquer processo conveniente entre os estados inicial e final. Fazendo a interpretação estatística da grandeza, ela pode ser dada pelo logaritmo neperiano do número de microestados possíveis, correspondente ao macroestado do sistema, multiplicado pela constante de Boltzmann, para ser expressa em unidades termodinâmicas (J/K). A Segunda Lei da Termodinâmica pode ser estatuída em termos da entropia, dizendo que a entropia de um sistema isolado nunca diminui, podendo permanecer constante nos processos reversíveis. Mas, é claro, pode diminuir nos sistemas não isolados. Aliás, é a possibilidade de redução localizada de entropia que possibilita a formação de estrelas, galáxias e o surgimento da vida a partir da matéria inanimada. Isso é possibilitado pela atuação de interações cumulativas, como a gravitação e as forças de Van der Waals.

Professor, acho que o senhor não entendeu aquela frase "A religião sem a ciência é cega. A ciência sem a religião é manca". O que Einstein quis dizer é que imaginar Deus criando o mundo como dita a religião é ignorância, enquanto que a ciência explicar o mecanismo do mundo sem seu agente, Deus, é en‎

De fato, imaginar o surgimento do Universo de acordo com cada mito religioso é totalmente inadmissível, não só porque eles não concordam entre si, como não se fundamentam em indício nenhum. Mas explicar o surgimento do Universo sem um agente provocador não e problema nenhum, pois isso é perfeitamente possível.

Professor, você falou que teria sido melhor para o Império Romano continuar pagão. Eu também diria que foi melhor para a França quando Napoleão a separou da Igreja. Você concorda?

Nesse aspecto sim. Mas eu sou um grande depreciador de Napoleão e de todos os generais conquistadores da História. São pessoas que não deveriam ter existido. Mesmo que elas possam ter feito algum bem em certo aspecto, no conjunto, toda iniciativa de dominação é nefasta. Nada pode ser imposto à força. Nem a civilização. Tem que ser um processo gradual de cativação e de educação dos povos para assimilarem os valores da civilização, preservando seus valores ancestrais de uma forma folclórica. O bom é que todas as civilizações façam um intercâmbio de seus valores, para que possam ser aproveitados por todas e o mundo inteiro se globalize, sem perder as características próprias de cada povo, cultivadas em harmonia com as benesses globais da civilização. Aparentemente o ocidente tem sido capaz de difundir seus valores no mundo todo. Mas há valores das civilizações indiana, chinesa, árabe, japonesa, indígena americana, polinésia e outras que são muito valiosos e deveriam ser acolhidos no mundo inteiro. Sem colonização militar, política, cultural, econômica e religiosa. Esse cinco aspectos, quando impingidos a um povo por outro que o domine são um verdadeiro desastre. Mas não quando assimilados de modo espontâneo, por adesão voluntária e pacifica. Não vejo necessidade, por exemplo, de manter os índios inculturados, caso não sejam. O que não acho é que isso deva ser impingido. E, uma vez aculturados, acho que têm que ser considerados cidadãos plenos e não incapazes. Ou seja, sujeitos a todos os ditames da lei que regula a vida de todo o mundo.

por que dizer que BIG BANG foi uma explosão é incorreto?‎

Não. O agnóstico pode achar que deus existe ou que não existe. O que o diferencia é que ele acha que isso não é possível de se saber. Por outro lado o gnóstico sabe ou que deus existe ou que não existe. E o cético acha que se pode saber, mas duvida desse conhecimento. Tanto o agnóstico, quanto o cético e o gnóstico podem ser ateus ou crentes. Eu sou um ateu cético. Pode haver também ateu agnóstico, ateu gnóstico, crente agnóstico, crente gnóstico e crente cético.

Como é a Física Matemática ? trabalha com o que ? E física estatística ?‎

Física Matemática, em verdade é Matemática. Tratam-se de tópicos de Matemática que possuem grande aplicação na solução de problemas físicos. Por exemplo: Equações Diferenciais Parciais, Cálculo Variacional, Cálculo Tensorial, Equações Integrais, Geometria Diferencial, Teoria de Grupos, Espaços Vetoriais Complexos, Análise Funcional, Formas Diferenciais, Funções Especiais, Álgebras de Lie, Geometria de Riemann, Transformadas Integrais, Séries de Fourier e outros assuntos.
Física Estatística é uma formulação da Termodinâmica a partir da Mecânica Clássica e Quântica de sistemas de muitas partículas por considerações estatísticas de seu comportamento.

Nunca consegui aprender física. E não é por falta de qualidade no ensinamento dos meus professores. É que, eu não consigo mesmo. Desde o nono ano (oitava série). O que devo fazer para aprender física mais fácil?

Para aprender Física o requisito fundamental é gostar de Física. Para tal você deve ler livros de divulgação de Física, assistir vídeos de experimentos de Física para ficar com vontade de saber a explicação do que é visto e, assim, se dedicar a estudar o assunto. Recomendo os livros:
Física em 12 Lições - Richard Feynman
Convite à Física - Yoav Ben-Dov
Fisica Conceitual - Paul G. Hewitt

O que você acha do casamento gay? Você acredita que DEUS fez o homem e a mulher e qualquer coisa diferente disso é obra do diabo?

O ser humano, como tudo o que existe no Universo não foram feitos por Deus, que não existe. Surgiram naturalmente. Diabo também não existe. Quanto às diversas orientações sexuais, assexualidade, bissexualidade, homossexualidade e heterossexualidade, são todas válidas e representam comportamentos normais na natureza. Eticamente nenhuma delas pode ser condenada, pois todas promovem a felicidade das pessoas e não prejudicam a ninguém. Caso a natureza fosse uma criação divina, isso faria parte dela, não sendo maldade de modo algum. A repugnância ao sexo como fonte de prazer que algumas religiões acolhem, bem como a condenação de qualquer comportamento sexual diverso dos padrões que estabelecem, é que é uma maldade.

Acha que um ateu deve ou não comemorar o natal?

Deve comemorar a ocasião de se confraternizar com as pessoas, de ser generoso e desprendido. Mas não deve comemorar o nascimento de Cristo como uma encarnação de Deus, pois ele, se de fato existiu, foi, simplesmente, um homem. A chamada "redenção" e uma das mais absurdas histórias que já foram inventadas.

Professor,Se tudo é formado por partículas(átomos) e átomos também são formados por partículas,por que o número não é infinito se cada partícula forma outra? Se pensar dessa forma que A forma B,C forma B e D forma C....... Caso não sendo infinito o que forma a última partícula?‎

Pelos conhecimentos atuais, todas as partículas da matéria são formadas de quarks e léptons. Esses por sua vez, são quantizações de campos, sendo o campo a entidade fundamental, que não é feita de nada mais primitivo. Há uma hipótese que considera que quarks e léptons sejam feitos de préons, mas não é confirmada. Outra hipótese, também não confirmada, é que esses préons ou os quarks e léptons diretamente, seriam feitos de anéis de cordas tubulares, cujos diversos modos de vibração diferenciariam o tipo de partícula. Essas "supercordas, por sua vez, também seriam quantizações de campos. Além das partículas materiais existem as partículas transmissoras das interações, os bósons, que são quantizações de outros campos também.

"A religião sem a ciência é cega. A ciência sem a religião é manca." Concorda? Justifique sua resposta rsrsrs‎

Não concordo. A ciência não precisa da religião para nada. Pelo contrário, a religião atrapalha a ciência. Pode-se dizer que a religião forneça um balizamento ético ao trabalho do cientista. Mas não é preciso religião para isso. A ética é humana, não religiosa. Por outro lado, se a ciência for se preocupar em conferir se o que ela descobre contraria ou não o que as diferentes religiões estabelecem, ela ficará completamente amarrada em sua atividade. Isso é inconcebível. A ciência tem que buscar as explicações que mais se aproximem da verdade, quer elas concordem quer elas discordem das religiões. Quanto às religiões, se elas forem atender o que a ciência diz a respeito de seus assuntos, elas se descaracterizarão. Para mim, ciência e religião são incompatíveis. Não há como conciliá-las.

Oque você acha sobre as diferentes religiões/crenças que tem pelo mundo?

São todas enganações. Elas consomem um grande tempo, dedicação, energia e dinheiro das pessoas, que poderia ser usado mais proveitosamente. Claro que elas fazem boas coisas, especialmente caridade e conforto mental. Mas o fazem com base em uma mentira e isso é muito mal. Elas dizem que existe um Deus e que ele premiará ou castigará cada um por seu comportamento nesse mundo, o que não é verdade. Então as pessoas agem bem para não serem punidas ou para serem premiadas. Isso não é uma boa coisa. As pessoas têm que agir corretamente e não fazer maldades simplesmente porque o bem é um alto valor em si mesmo e o mal algo muito ruim, independentemente de qualquer castigo por ações más ou prêmio por ações boas. É assim que se tem que educar as pessoas. Além do mais, a esperança de uma bemaventurança eterna costuma inibir a luta pelo prevalecimento do bem e pela erradicação do mal na vida cotidiana. A sociedade tem que ficar vigilante e coibir o mal e não ficar deixando para "Deus" fazer isso, pois não será feito.

Professor,A aceleração de um carro pode ser variável?Se sim, existe alguma equação para prever qual a sua posição do carro em função do tempo?‎

Claro que pode. Se você souber a função a = a(t), você pode integrar duas vezes. A primeira integral dá a velocidade em função do tempo: v = v(t) = ∫ a(t)dt . A segunda integral dá a posição s = s(t) = ∫ v(t)dt

Professor, se Deus não existe, qual a razão de vivermos? Observando que muitos já nascem sofrendo‎

A única razão da vida é a própria vida. Vivemos para curtir a vida. Só isso. A vida é de uma preciosidade ímpar. Um privilégio glorioso para qualquer vivente. Viver a vida de forma significativa, de tal modo que se sinta satisfeito e feliz por se estar vivendo é a razão de se viver. E isso se alcança quando se dedica a fazer o bem e a ser virtuoso. Não há nenhuma necessidade de se invocar nenhum Deus para dar significado à vida. É preciso que nos dediquemos a construir uma sociedade em que ninguém sofra por razão nenhuma. Em que todos tenham tudo o que precisam e, até, muito do que desejam. Sem que nada obste a felicidade de ninguém. Essa é uma ótima razão para se dedicar a vida e conferir a ele um grande significado.

Professor, acha que é importante para um Ateu requerer formalmente à Igreja que seja considerado um apóstata? Estive lendo este curto artigo, e me pareceu coerente: http://apostasiacoletiva.wordpress.com/por-que-apostatar/ Gostaria da sua opinião. Obrigado

Não acho que seja importante não, mas pode. Isso advém não de minhas concepções ateístas mas de minhas concepções anarquistas, segundo as quais tudo deve ser desformalizado. Ou seja, que não se tenha registro de nada do que você seja. Para mim nem deveria haver certidão, carteira de identidade, nada disso. Nem sobrenome as pessoas deveriam ter. Só nome. Uma coisa completamente irrelevante é saber de que família a pessoa seja. A família é a humanidade toda. Nada de contratos. Numa sociedade de altíssimo nível de civilização, em que não haja dinheiro e nem propriedade e, portanto, em que não haja herança, nada disso tem a menor relevância.

O senhor já observou em sua carreira vários professores, que fazem questão de serem 'carrascos' com aluno, pelo simples fato de ser? Isso seria frustração ou outra coisa? qual a sua opinião?

Isso acontece e é muito triste. Não é uma frustração. É uma espécie de vaidade, de vontade de se mostrar como mais capaz. De querer se afirmar. De não aceitar que outros possam ser tão bons ou melhores do que ele. Em suma, uma falta de segurança em seu próprio valor. Quem é seguro de si não precisa demonstrar nada, especialmente espezinhando os outros para se glorificar. Trata-se de um distúrbio de personalidade e uma completa falta de caráter que precisa ser contida pelas coordenações e pelas direções dos colégios. Pessoas assim não podem ser professoras.

Ah, eu sou apaixonada por isso. Amo estudar. Quero ter a oportuniade de lecionar matérias específicas de Astrononia/Cosmologia e, certamente, Física. Nenhum assunto me emociona mais. Só tenho medo de não ser boa o suficiente por, às vezes, perder-me nas palavras. Espero que com a prática eu melhore.‎

Se é assim, então você consegue. Para treinar, comece a escrever textos de física para ensiná-la para pessoas leigas em física. Não para alunos de escolas, mas para adultos que nunca estudaram física. Aí você vai ter que se virar para conseguir explicar, para colocar em palavras uma forma inteligível de aprender o assunto. Faça um blog e vá escrevendo. Pegue temas e os desenvolva.

o que acha da maconha professor?

Nada bom. Todas as drogas psicotrópicas, bem como o tabaco e o álcool são substâncias perniciosas para a saúde física e mental. Absolutamente não recomendo o seu consumo, seja por que razão for. Apenas o álcool, em pouca quantidade, pode ser aceito. O resto é uma droga mesmo. Não compensa o prazer que é capaz de propiciar no momento, de forma nenhuma.

Professor, eu começarei a cursar Física, mas não sei exteriozar perfeitamente bem o que se passa em minha cabeça. Acredito que com a prática vou conseguir, o senhor concorda? Para a carreira docente e o bom ensinamento do conteúdo a prática basta, sem essa característica estar intrínseca em mim?‎

Não. Não basta. Para ser um bom professor é preciso que a pessoa seja fascinada, deslumbrada, entusiasmada com o assunto que leciona. Tem que gostar e querer saber a respeito tudo que for possível, muito além do que é preciso para ensinar. Tem que viver sua matéria em todos os momentos, não só quando está trabalhando. Se a Física não te fascina, não te emociona, talvez seja o caso de você achar o que te faz sentir assim e mudar. É o que penso.

Acho Tesla mais visionário/inteligente que Einstein. Qual é a sua opinião?

Cada um tem um aspecto diferente de inteligência. Tesla era um engenheiro cientista, mas uma pessoa empresarial, de visão prática, mesmo que idealista. Einstein não era um cientista aplicado. Era um teórico sem a menor característica empresarial. Não se importava com a utilidade do conhecimento. Ambos, contudo, tinham uma grande intuição, fator que diferencia a genialidade do grande talento apenas. Em todas as áreas, a criatividade não depende apenas da inteligência, mas de algo mais em termos de intuição, de raciocínio inconsciente. De ousadia.

O que o senhor acha sobre alguém se graduar em Engenharia Aeroespacial, e depois fazer a pós de Astrofísica? Quais as áreas que poderei trabalhar?

Esses são campos bem distintos de atuação. O engenheiro aeroespacial projeta foguetes, satélites e naves e o astrofísico investiga a estrutura e a dinâmicas de estrelas, galáxias, nebulosas. O primeiro é um cientista aplicado e o segundo um cientista puro. Isto é, Astrofísica é uma ciência sem aplicações práticas de utilidade nenhuma. O trabalho de um astrofísico geralmente acontecerá em universidades e institutos de pesquisas, pois a indústria não se interessa por nada que não produza retorno financeiro nenhum. Nas agências espaciais há lugar para os dois, pois muitas sondas são projetadas para pesquisar dados astrofísicos, como a coroa solar, os outros planetas e suas luas. Então o astrofísico tem que trabalhar junto ao engenheiro para projetar os sensores das sondas.

Existe uma certa "rivalidade" entre físicos e matemáticos?‎

É o que parece. Muitos físicos acham que os matemáticos puros se ocupam de assuntos sem relevância nenhuma, mas muito complicados, para exibir vaidosamente a sua inteligência. Por outro lado os matemáticos acham que os físicos usam a matemática sem saber a razão de ser do que estão usando. Isso é uma grande bobagem. Historicamente, em geral, matemáticos eram físicos e físicos eram matemáticos, além de filósofos. Descartes, Leibiniz, Newton são grandes exemplos. Físicos também se ocupam de assuntos sem aparente utilidade, como a Cosmologia e a Teoria Quântica de Campos.

O que é um insight?

Insight é uma inspiração, isto é, uma intuição a respeito de algo que se esteja pesquisando. Uma ideia que se tem, aparentemente sem fundamento. Em verdade, trata-se do resultado de algum raciocínio inconsciente, em que a mente trabalhou sobre todos os conhecimentos disponíveis sem que a pessoa estivesse consciente desse pensamento e o apresentou à consciência. Como o modelo da dupla hélice do DNA que Watson descobriu.

Professor,pessoas que sabem português são menos inteligentes dos que as que sabem matemática?

Não são não. Não necessariamente. Podem ser ou não ser. A inteligência tem vários aspectos. Pode-se ser inteligente em um ou outro deles. Ou em mais de um. Alguém pode ser inteligente tanto no aspecto linguístico quanto no lógico-matemático.

Bem professor já que respondido sobre minha pergunta anterior,eu tenho um última,venho sendo muito cobrado para trocar para engenharia mecânica e queria saber se em questão de salário a diferença chega a ser muito com relação aos empregos relacionados a Física-Bacharelado?

Jamais se decida por sua profissão em função do rendimento. Você vai ser infeliz a vida toda. Faça o que gosta. Um engenheiro pode ganhar mais ou menos do que um físico. Vai depender de ser um ótimo engenheiro ou um ótimo físico. Quem for bom mesmo no assunto a que se dedicar sempre vai conseguir um bom rendimento. Então escolha o que gosta e se esforce para ser um excelente profissional. Não daqueles que se contentam com o mínimo necessário para dar conta, mas daqueles que vão muito além do que é preciso. Esses são insubstituíveis e não precisam procurar trabalho. São procurados.

Professor, as únicas coisas que me interessavam no colégio eram física em geral, química em geral e ecologia e vida animal, em biologia. De resto, nada me interessa. O que me aconselha em relação à profissão? Que faculdade posso fazer? (Se eu não me interessar em lecionar)‎

Sugiro que você faça Física, porque, como físico, você pode, também, enveredar por outras considerações, como Ecologia, mais facilmente do que, como biólogo, enveredar pela Física. Você vai ter que fazer bacharelado, mestrado e doutorado para ser um cientista. Mas eu recomento que faça, também, licenciatura. Porque todo cientista, em geral, dá aulas. E é péssimo um professor que não saiba lecionar direito, mesmo que seja ótimo no conteúdo. Só que a Física de verdade, infelizmente, é bem diferente daquela que estuda no colégio. Há mais de quarenta anos que eu me bato, em vão, para que a Física Moderna seja abordada no Ensino Médio. Só pude ter o prazer de lecionar relatividade, física quântica, física atômica, física nuclear, física estatística, física do estado sólido em uma escola nesse Brasil. A EPCAR, em Barbacena. Mas lá, eles nem queriam saber o que ia cair no vestibular.

Como motivar as pessoas no anarco-comunismo, para que elas trabalhem?

Mostrando que é pelo trabalho de todos, uns pelos outros, que a sociedade consegue prosperar como um todo, de modo a beneficiar a todos. Isso é uma questão de conscientização que se tem que trabalhar num processo educativo ao longo de muitas gerações. Não é com as pessoas que estão no mundo de hoje. O anarquismo é, principalmente, um processo de mudança de cosmovisão. Uma opção pela cooperação em lugar da competição, pelo altruísmo em lugar do egoísmo, pelo trabalho em lugar da ociosidade, pela generosidade em lugar da ganância. Para se chegar a uma sociedade ácrata é preciso mudar o ser humano. Isso vai custar dezenas de gerações. A motivação é o ideal de se viver numa sociedade perfeita, pelo que se é capaz de fazer muito sacrifício.

Olá professor,Eu amo física,sou ateu e quero fazer bacharelado em física na UFCG, gostaria de saber se compensa?e em questão de trabalho?‎

Se você se destacar na turma e fizer mestrado e doutorado, certamente conseguirá passar em algum concurso para professor ou pesquisador de universidades públicas ou institutos de pesquisa. Neste caso seu salário, com as participações em projetos de pesquisa, poderá chegar aos trinta mil reais por mês. Mas vai começar com bem menos, uns quatro ou cinco mil por mês. Você também poderá lecionar em faculdades particulares ou escolas e cursinhos. Se for em São Paulo e você for um professor muito bom mesmo, pode ganhar uns vinte mil reais por mês. Mas se você for apenas razoável, não consegue isso não. Você também poderá escrever livros didáticos, que dão muito dinheiro. Mas o trabalho é árduo. Para ser aceito por uma editora, tem que ter características muito especiais. No entanto, mesmo que você não vá ser um expoente de sua profissão, penso que dá para viver bem como professor e pesquisador. E se isso for o que você gosta de fazer, claro que vale a pena, pois você viverá feliz, nesse aspecto.

gás é matéria pouco condensada?

Sim. Gás é matéria em que a separação entre as moléculas ou átomos é maior do que o tamanho deles, de modo que não se tocam, exceto nas colisões devido a seu movimento. Em sólidos e líquidos as moléculas ou átomos se prendem uns aos outros. A diferença é que nos sólidos essa ligação é bem rígida e nos líquidos ela permite que as partículas rolem umas sobre as outras, permitindo o escoamento. Os gases são expansíveis, sempre ocupando o máximo volume permitido pelo recipiente ou pelo campo gravitacional, no caso de gases sem paredes, como a atmosfera e as nebulosas. Assim os gases são muito compressíveis, podendo diminuir de volume até que suas moléculas ou átomos se toquem, quando ele se liquefaz ou se solidifica, dependendo da temperatura e da pressão.

Certa vez ouvi dizer que vidro é líquido. E quem falou isso foi um professor de física. Tem alguma coerência?

Sim. O vidro é, estruturalmente, um líquido de altíssima viscosidade. Suas moléculas não são rigidamente presas a seus lugares, como em um sólido verdadeiro, dito cristalino. Elas podem rolar umas sobre as outras e assim o vidro escorrer. Ao se aumentar a temperatura, a mobilidade das moléculas aumenta e o vidro vai amolecendo. Em um verdadeiro sólido, como o gelo, o aquecimento provoca o desprendimento de moléculas da superfície, que formam a água líquida, mas o resto que sobra continua sólido sem amolecer. Mas o vidro pode quebrar. Isso também acontece com outros materiais, como o lacre, o piche e alguns outros.

Se o fogo não pertence a nenhum estado físico ele é o que?‎

Fogo é um plasma, o quarto estado da matéria. Trata-se de um gás ionizado e a tal temperatura que fica luminoso. Só isso. No fogo existem dois sistemas: o dos íons e o dos elétrons livres. Como a massa dos elétrons é bem menor do que a dos íons, à mesma temperatura, eles se movem a velocidades muito maiores. Mas são os íons que emitem luz. O gás de elétrons é invisível.

professor por que para o senhor o ciumes é uma forma de desamor e falta de confiança?‎

O ciúme (e não "ciúmes") é uma grandessíssima prova de desamor e de egoísmo. Pode envolver falta de confiança, mas isso, também, é prova de desamor. Quem ama confia e quer o bem da pessoa amada e não o seu domínio, o seu controle, a sua posse. O ciumento é um egoísta. um caráter malsão. O verdadeiro amor é totalmente altruísta. Isto é, quer a felicidade da pessoa amada, mesmo que essa felicidade consista em que ela viva um amor com outra pessoa. Quem ama que o seu amado ou amada completamente livre. Quem ama não vigia, não controla, não proíbe nada, não toma conta. Só cuida do bem de quem ama.

Sobre intuição ou sexto sentido: Você confia na sua intuição ou sexto sentido? Acha prudente tomar decisões baseadas nela?‎

Intuição é, simplesmente, raciocínio inconsciente. Se você é uma pessoa com grande bagagem de conhecimento sobre múltiplos assuntos e alguém acostumado a pensar, argumentar, raciocinar, então seu inconsciente está bem preparado para fazer isso sem o seu conhecimento. Daí ele lhe propõe soluções, a partir de suas operações, que consistem em sua "intuição". Vale a pena confiar na intuição para a tomada de decisões. Eu sempre me decidi muito mais pela intuição do que pela razão explícita. A intuição sempre apresenta uma proposta mais condizente com a sua felicidade. Mas, muitas vezes, contrária a seu sucesso ou prosperidade. Todavia o sucesso e a prosperidade são muito menos importantes do que a felicidade. Só não deve confiar na intuição a pessoa carente de conhecimentos e habilidades mentais. Porque se ela é assim em modo consciente, também o será inconscientemente.

Pretendo cursar astronomia, pós graduação em biologia e então partir pra astrobiologia, acha um campo promissor pra trabalhar ??? Tenho 17 anos

É preferível fazer o oposto. Cursar Biologia e pós-graduação em Astronomia. Mas Astronomia requer muito conhecimento de Física e Matemática. Em geral quem faz Astronomia faz graduação em Física ou Matemática e depois mestrado em Astronomia. Qualquer que seja a sua opção entre a graduação e a pós-graduação você vai ter certa dificuldade de adaptação. Se for fazer o mestrado em Biologia, tendo feito graduação em Astronomia, vai ficar difícil. O campo da Astrobiologia é interdisciplinar e quase exige que a pessoa faça as duas graduações. Tem que conversar com quem mexe com isso e descobrir onde encontrar pessoas assim. É um campo muito interessante de pesquisa, mas não conheço onde ele seja feito no Brasil. Você vai ter que pesquisar na internet.

A soma dos ângulos internos de um triângulo só da 180 quando o espaço que ele está não esta curvo certo? quais as outras exceções? quais os valores da soma em um espaço curvado positivamente e negativamente? Grato

No caso de espaços curvos a soma depende da curvatura do espaço e do tamanho do triângulo. Imagine um triângulo curvilíneo formado por um arco do equador terrestre e dois meridianos separados por uma diferença de longitudes de 90°, que se interceptam no pólo. Nesse caso a soma dos seus ângulos internos será 270°. Num espaço de curvatura negativa será menos do que 180°.

" A lua não cai na terra pois ela nunca acha o chão", " A velocidade tangencial da lua é maior que a aceleração voltada para baixo, por isso ela mantem sua orbita ". " A lua descreve uma linha reta em um espaço curvado pela terra" '-' Qual delas afinal usar para descrever essa situação???

Você nunca pode dizer que a velocidade tangencial da Lua seja maior, menor ou igual a sua aceleração centrípeta, pois velocidade e aceleração são grandezas de espécies diferentes e, portanto, não comparáveis. O que acontece é que a Lua se situa em uma distância tal que o campo gravitacional da Terra lá coincide com o valor da aceleração centrípeta que ela precisa para ficar em órbita com a velocidade tangencial que ela possui. As três descrições são corretas e equivalentes. Ou como um movimento circular uniforme, ou como um movimento de queda livre ou como um movimento geodésico inercial em um espaço-tempo curvo.

É verdade que o físico sabe mais matemática do que o matemático?‎

Depende da pessoa. Pode saber. Em geral o matemático sabe mais de um aspecto específico em que aprofundou seus estudos. O físico teórico é obrigado a saber muitos tópicos de Matemática, de modo bem abrangente a aprofundado, que o matemático, em geral, não cuida de dominar.

A física pode explicar a origem do universo ?‎

Pode, mas ainda não conseguiu. A teoria do "Big Bang" explica a expansão do espaço continente do conteúdo do Universo, considerando que ele já existisse, inicialmente de forma extremamente concentrada, de modo que todo o Universo Observável (atualmente com um diâmetro de 92 bilhões da anos-luz) ocupasse um volume menor do que um elétron. Todavia não se sabe como se deu o surgimento desse conteúdo, cujo espaço continente passou a expandir. Isso não autoriza a supor, sem verificação, que tal conteúdo tenha sido "criado", sem ter do que proceder, por alguma entidade extrínseca ao Universo

A luz tem massa?

Não. A luz e todas as formas de radiação eletromagnética possuem energia. Essa energia, para efeitos inerciais e gravitantes de algum sistema que contenha radiação, equivale a um total de massa dado pela equação E = mc². Mas não significa que a luz tenha massa, pois massa é uma grandeza que mede a quantidade de inércia e a "carga" gravitacional de um sistema quando "em repouso". E a radiação não existe em repouso.

É correto considerar que se e=mc² então eu posso generalizar que Massa = E/C² ? Em todos os casos da física? e essa energia intrínseca aos corpos se manifesta de que forma? digo, a lua tem uma massa, toda a energia dela é cinética

Essa equação se aplica quando há uma transformação de matéria em radiação, de modo que a massa da matéria e a energia da radiação que se transformaram uma na outra ou vice versa se relacionam por essa expressão. Ela também fornece o equivalente em massa para o efeito gravitante de certo valor de energia. Isso não significa que os fótons, que são os quanta de radiação emitidos e absorvidos por sistemas, possuam massa, já que massa é uma grandeza que mede um atributo inercial e gravitante de sistemas "em repouso". Radiação não existe em repouso.

Professor ultimamente tenho me irritado facilmente com pessoas burras. Pessoas que se equivocam, tem atitudes bobas, que não pensam antes de falar ou fazer algo. Não tem bom senso. O pior, é que meu trabalho envolve o "povão" mesmo. Que conselho você me daria?‎

Que se contenha nisso. Ninguém é burro porque quer. Não é culpa da pessoa. Mesmo a ignorância. É preciso tolerar a burrice e a ignorância com caridade e sem menosprezar, ridicularizar nem espezinhar o burro e o ignorante. Pelo contrário. É preciso ajudá-lo a deixar de ser burro e ignorante. Assim é que se revela uma nobreza de caráter, que é mais importante do que ser culto ou inteligente.

Na sua opinião qual a melhor maneira de estimular a inteligência e aprender mais coisas em menos tempo??

Vencer desafios. Complicar a vida. Dificultar tudo o que for fazer. Assim é que se estimula a inteligência. Usar o relógio invertido. Andar de fasto. Trocar roupa de olhos fechados. Escrever da direita para a esquerda. Aprender novos idiomas. Aprender instrumentos musicais. Aprender novos esportes. Quanto mais complicado, melhor. Comer com a mão que não é a dominante. Resolvendo enigmas. Ler e escrever bastante também estimula a inteligência. Procure artigos e livros sobre "neuróbica". Você aprenderá mais coisas ao mesmo tempo simplesmente se dedicando a seu aprendizado com muito empenho.

É uma vergonha que o nosso país não tenha feito ainda nenhuma exploração espacial. Primeiro de tudo não temos sequer uma estação de lançamento de foguetes e segundo quando é lançado um satélite para o espaço algo dá de errado como mostrado no site apolo11.com.

Não acho que seja vergonha, pois, a exceção de uma meia dúzia, nenhum país do mundo fez exploração espacial alguma, alguns até mais desenvolvidos do que o Brasil.

como é possível saber a composição química das estrelas?

Pela análise do espectro da luz que elas emitem. Cada elemento químico possui um conjunto de linhas de emissão, correspondentes a sua distribuição eletrônica, que podem ser identificadas e revelar a presença do elemento. A intensidade das linhas revela a proporção daquele elemento no conjunto de todos. Outros indicadores são a densidade da estrela e sua temperatura.

Olá, professor, tudo bem? Quero fazer Artes Visuais e quando comentei isso com o meu professor de Física, ele começou a falar que artes é inútil e disse que a maioria das pessoas que fazem Exatas pensam que nem ele. Gostaria de saber se você também acha isso ou não. Abraços!

Não penso que se dedicar à arte seja uma atividade irrelevante. Pode-se dizer que a arte é inútil se se tiver um conceito restrito do que seja utilidade, fixando-se apenas no aspecto de algo sem o que a vida se inviabiliza. Sem arte é possível se viver, não há dúvida. Assim a arte seria inútil. Mas, numa visão mais abrangente do que seja utilidade, pode-se ver que a arte propicia uma qualidade de vida com alegria e satisfação que sem ela não seria tanta. Assim a arte é útil para dar sentido e felicidade à vida. O mesmo acontece com certar áreas da ciência. Cosmologia, por exemplo. Se não se souber nada a respeito, a vida não sobre alteração nenhuma. Mas o homem tem curiosidade sobre como é o Universo e a Cosmologia supre isso. Já pensou a vida se não houvesse música, poesia, cinema, teatro, literatura e as artes plásticas? Não seria inteiramente insossa? Não é verdade que a maioria das pessoas que se dedicam a ciências exatas tenha uma opinião depreciativa sobre a arte. Só que tem a mentalidade tacanha.

É correto afirmar que força é o produto de uma interação que causa alteração no momento linear de um corpo???‎

Força é a grandeza que mede a intensidade de uma interação. O que causa alteração no movimento de um sistema e a interação. O movimento do um sistema pode ser especificado por grandezas que o medem, como o momento linear e a energia cinética. A interação, atuando ao longo do tempo e do espaço, provoca mudanças nessas grandezas, cujos valores estão ligados a grandezas que medem a quantidade de interação, que são o impulso e o trabalho. É preciso não confundir as entidades e as ocorrências com seus atributos e as grandezas que os medem. Força é uma definição arbitrária que poderia ser diferente. Não é a força que causa nada, é a interação. A força é só um modo de especificar o quão intensa seja a interação.

Ola professor, preciso de um conselho, estou namorando a um pouco mais que um mes, mas meu namorado não quer conhecer meu pai, nao sei o porque dessa atitude, mas ele me cobra muito o sexo, o que o Sr. Acha? Muitos dizem q é so medo do meu pai e outros dizem que se ele me amasse ja tinha ifo la,

Se ele te cobra muito sexo e não quer conhecer o seu pai, eu acho que ele está querendo você só para fazer sexo e não como uma namorada para amar de fato. Não acho que o sexo deva ser evitado, mas acho que o namoro tem que ir além disso. Tem que envolver dedicação, querer bem, compartilhamento, cuidado, ternura, amizade. Inclusive privação do sexo. E, certamente, interesse pela pessoa como um todo, o que inclui conhecer a sua família.

para algo se mover....precisa de uma forca inicial, Certo? queria entender como pode o vento se "mover" levando em conta que nada semprr existiu. Tipo. tava sentado almocando...quando o vento me bate...ai....pensei sobre...como?‎

Força só é necessária para alterar o movimento. Sua manutenção é feita pela inércia. No caso do vento, ele é provocado pelo aquecimento do Sol maior ou menor em lugares diferentes, o que provoca a dilatação do ar mais aquecido, que empurra o ar vizinho, provocando o vento.

Professor, estava lendo um livro, neste livro vi que todas as imagens de Adão, ai pensei, Se Adão e Eva eram brancos, de onde vieram os Negros?‎

De acordo com a Bíblia, os negros, bem como os indianos e os chineses são descendentes de Cam, um dos filhos de Noé. Os árabes e judeus são descendentes de Sem, outro filho e os europeus de Jafé. A cor negra da pele foi uma maldição de Deus a um dos filhos de Cam, por ter debochado de Noé.

Professor, qual a diferença e semelhança, se existir, entre campo elétrico e campo magnético? Sempre acabo me confundindo com os dois‎

O campo elétrico é produzido por e faz força sobre qualquer carga elétrica, enquanto o magnético só é produzido por e faz força sobre cargas em movimento.

Caro Ernesto, quais fatores devem ser considerados e quão relevantes eles são na escolha de uma profissão?

Primeiro: Se se trata de uma ocupação que a pessoa goste de fazer. Que lhe dê satisfação e prazer na vida. Que confira significado à sua vida.
Segundo: Se se trata de uma ocupação que promova o bem geral da população. Que tenha grande utilidade pública. Que faça o mundo se tornar melhor.
Terceiro: Se se trata de uma ocupação que permita à pessoa um nível de prosperidade que a faça levar a vida sem apertos e com conforto.

Quando faço a questão pressupondo que o sentido da corrente está no sentido horário, meu resultado bate com o resultado dele. Quando parto do sentido anti-horário, o resultado fica negativo. No vídeo, onde um professor resolve, ele diz que o resultado tem que ser o rigorosamente o mesmo. Link

Ao resolver um problema de circuito elétrico pelas leis de Kirchoff e o resultado de alguma corrente der negativo é porque a escolha do sentido dela foi a escolha errada. Mas isso não tem importância. Qualquer que seja a escolha, o resultado será correto, pois, então, você inverterá o sentido da corrente.

O que o senhor pensa a respeito do suicídio?‎

Válido, mas não recomendável. Um direito de cada um. Todavia considero um grande equívoco, já que a preciosidade da vida é inestimável. Só mesmo em caso extremo.

Prof, o senhor frequentemente recomenda alguns links do wikipedia. O que eu gostaria de saber, é se o senhor não acha esse site pouco confiável, visto que qualquer pessoa pode alterar os conteúdos

Mesmo que os artigos da Wikipedia possam ser mudados por qualquer um eu faço muito uso deles. Para ter mais confiança eu comparo os textos em português, inglês, francês, italiano e espanhol, que posso ler e entender. O que coincidir, muito provavelmente estará certo. Além do mais, especialmente em inglês, os artigos costumam ser recheados de referências, nas quais se pode conferir o que é dito. Acho ótima a Wikipedia. Dá para perceber quando algo foi colocado de forma intencionalmente errônea.

O que é estado laico?‎

Aquele que não privilegia nenhuma religião. Isto é, que, em suas leis, não leva em consideração o fato do cidadão ser filiado a qualquer religião que seja. Que não toma partido a favor de nenhuma delas, bem como não discrimina nenhuma delas. O Brasil é um estado laico. No entanto, paradoxalmente, privilegia o cristianismo com a presença de crucifixos em locais públicos, como o salão do juri do Supremo Tribunal Federal. Isso é inconstitucional e jamais poderia ser admitido, justamente naquela casa. Outro fato que arranha a constituição é a frase "Deus seja louvado" que consta das cédulas do dinheiro brasileiro.

Cabelos longos é uma das formas mais fáceis da pessoa se diferenciar do comum e se afirmar como diferente. Ou barba longa, também. Geralmente, as pessoas que se dedicam a atividades incomuns na sociedade, como artistas, cientistas, filósofos, gostam de mostrar que não são pessoas comuns, que sua atividade não é uma atividade usual. Então exibem alguma característica que visualmente as distinga, como cabelo grande ou roupa incomum. É o que acontecia com as batinas dos padres e sua tonsura. Nem todos, contudo, agem assim. Eu mesmo já tive uma fase desse tipo. Depois concluí que é bobagem. http://wolfedler.blogspot.com.br/2009/03/blog-post.html

Qualquer que seja ela, se considera que satanás existe, está errada. A concepção de LaVey, no entanto, não considera que exista Deus nem Satã como seres espirituais reais, mas que Satanás seja apenas um símbolo da natureza inerente do homem. Mesmo assim, acho uma grande bobagem, exatamente por erigir suas crenças em uma "Bíblia Satânica" e suas práticas em uma "Igreja Satânica". Acho que até seja admissível quem siga esse tipo de filosofia. A questão é que fica engessada e não se abre para o ceticismo e o livre pensamento que são essenciais ao ato de filosofar.

existe alguma explicação sobre muitos artistas terem cabelos longos? não só artistas mas pessoas que fogem do padrão social. sempre observo em vídeos, até mesmo na rua, senhores e jovens que tocam musica clássica, erudita, metal, entre outros gêneros. não só em gêneros musicais mas em outros aspect

Cabelos longos é uma das formas mais fáceis da pessoa se diferenciar do comum e se afirmar como diferente. Ou barba longa, também. Geralmente, as pessoas que se dedicam a atividades incomuns na sociedade, como artistas, cientistas, filósofos, gostam de mostrar que não são pessoas comuns, que sua atividade não é uma atividade usual. Então exibem alguma característica que visualmente as distinga, como cabelo grande ou roupa incomum. É o que acontecia com as batinas dos padres e sua tonsura. Nem todos, contudo, agem assim. Eu mesmo já tive uma fase desse tipo. Depois concluí que é bobagem.
http://wolfedler.blogspot.com.br/2009/03/blog-post.html

Então como o senhor explica a inexistência do destino?

Não precisa explicar. O que precisaria explicar seria a sua existência. Uma vez que não há razão nenhuma para haver destino, ele não existe. O que seria destino? Alguma programação cósmica que determinasse o que fosse acontecer no futuro. Ora, o desenrolar dos acontecimentos é fruto de uma série de injunções que se dão entre os inúmeros eventos que estão sempre acontecendo. Há eventos que influenciam os outros, bem como há eventos que ocorrem aleatoriamente, sem que tenham sido provocados por nada. Mesmo no caso dos que são provocados, os provocadores não funcionam deterministicamente, isto é, eles podem acarretar uma coleção de possibilidades de efeitos, sem que se possa prever o que se dará. Isso tudo é interligado por um intrincadíssima rede completamente imprevisível. Ou seja, se se soubesse, num dado momento, tudo o que estivesse acontecendo com tudo o que existe no Universo todo, nem assim seria possível prever como seria o momento subsequente. Isso contraria totalmente a concepção de Laplace: "Une intelligence qui, pour un instant donné, connaîtrait toutes les forces dont la nature est animée, et la situation respective des êtres qui la composent, si d’ailleurs elle était assez vaste pour soumettre ces données à l’Analyse, embrasserait dans la même formule les mouvements des plus grands corps de l’univers et ceux du plus léger atome : rien ne serait incertain pour elle et l’avenir, comme le passé serait présent à ses yeux.". A razão está na incausalidade e no indeterminismo intrínsecos da natureza, como demonstrado pelas concepções quânticas, sobejamente provadas que são o modo de agir da natureza. Em suma: Não há nenhuma possibilidade de que o futuro seja conhecido. Logo, destino não

"Talvez em um futuro onde a tecnologia e a robótica estejam tão avançadas a ponto das máquinas fazerem tudo por nós a anarquia seja possível" Concorda professor?

Não precisa disso. As pessoas podem fazer tudo elas mesmas, umas pelas outras. Isso, de uma forma comunitarista, é a melhor maneira de otimizar todos os procedimentos para a obtenção dos melhores resultados com os menores esforços. O compartilhamento é várias vezes mais eficiente e eficaz do que o individualismo e a competição. Isso é algo a que as pessoas podem concluir sem dificuldade, desde que abdiquem de pretenderem ter vantagens em relação às demais.

O que você acha da teoria da atração? Ou Karma, se preferir. Como eu já prevejo à sua resposta, por que é algo falso?

Exato. Minha resposta é de que seja uma completa besteira. Teoria da Atração e Karma são coisas distintas. Teoria da Atração é a concepção de que a mente possa interferir nos acontecimentos do mundo. Não pode, de modo nenhum. Não existe nenhum mecanismo de atuação da mente diretamente sobre o mundo objetivo fora dela. Só por meio dos comandos da fala e dos movimentos é que a mente pode se comunicar com o mundo, isto é, através do corpo e do que o corpo seja capaz de fazer. Karma seria o fato de que tudo de bom e de ruim que a pessoa fizesse ficaria gravado e a próxima pessoa em que sua alma encarnasse sofreria por isso. Ora, como alma não existe e nem reencarnação, karma não existe.

quarta-feira, 12 de fevereiro de 2014

O que acha de liberarem o Topless do Rio? E como a mídia mostra cada vez mais pessoas quase nuas em horario nobre e estamos perto de termos cenas de sexo antes da meia noite?

Não vejo problema em liberar o Topless nas praias de qualquer lugar do mundo. Ou, até mesmo, o nudismo, depois de se acostumar com o Topless um certo tempo. Só não acho conveniente na cidade, por questões até de higiene e estética, bem como de respeito. Quanto a cenas de sexo, sinceramente, não vejo porque sexo deva ser um tabu. É uma atividade normal do ser humano. Qual o problema de ser mostrado? Porque crianças não podem saber que adultos fazem sexo? Não consigo entender isso mesmo. Para não tirar a inocência delas? Mas inocência é algo bom? Dizem que elas vão por maldade. Que maldade existe no sexo? Para mim é bondade. Claro que acho que é preciso fazer uso do sexo com responsabilidade e maturidade. Mas isso as crianças podem aprender, de modo que não comecem a fazer sexo muito novas, o que pode ser prejudicial a seu desenvolvimento psíquico. Acho que sexo teria que ser um assunto tão comum nas conversas como gastronomia. E falado sem risadinhas, na maior naturalidade. Em rodas de homens e mulheres misturados.

Então eu iria cozinhar por amor, organizar revistas em uma banca por amor, lecionar por amor, recolher o lixo municipal por amor, pavimentar as ruas por amor? E as pessoas interesseiras, oportunistas e de ego inflado? Não creio que a humanidade esteja pronta pra isso, nem eu.‎

Realmente a humanidade não está pronta para isso. Por isso é que dito que a anarquia ainda levará séculos ou milênios para acontecer. Mas é preciso que se vá fazendo um trabalho educativo para que as pessoas se conscientizem de que é assim que deve ser e que deixem de ser interesseiras, oportunistas, egoístas, gananciosas, preguiçosas e se tornem altruístas, generosas, desprendidas, bondosas, diligentes e virtuosas de modo geral. Isso é um trabalho de educação do caráter que precisa ser urgentemente implementado nas escolas, inclusive com reprovação por mau-caratismo.

Professor, Comunismo é a distribuição igualitária da miséria. Concorda com essa frase? Excelente noite pra você

De modo nenhum. Pode ser só onde não houver riqueza. Mas o Comunismo é o compartilhamento dos bens pela sociedade como um todo. E esses bens podem e devem ser abundantes. Note que comunismo não é o que existe em Cuba e na Coréia do Norte e nem o que existiu na antiga União Soviética e seus satélites. Aquilo é Socialismo de Estado, que é muito diferente, mas foi chamado, erroneamente, de Comunismo. No Socialismo de Estado os trabalhadores são todos empregados de um único patrão, o Governo. No Comunismo não há empregados. Todos os trabalhadores trabalham para si mesmos e se auto gestam. Pode até haver algum governo para coordenar os projetos de maior abrangência, mas nem precisa ter. Esses projetos podem ser geridos por comitês "ad hoc", das comunidades envolvidas. Como a construção de uma longa ferrovia, de uma usina hidroelétrica ou atômica e empreendimentos de grande vulto.

Professor, outra pergunta, o que seriam atividades desnecessárias para a vida? E se tudo fosse de todos e a competitividade se extinguisse, o progresso não iria se extinguir também? A sociedade não cairia em estagnação?‎

Atividades desnecessárias são as que se não se relacionam diretamente com os fins e sim com os meios dispensáveis. Por exemplo, todas as atividades relacionadas com dinheiro, como atividades bancárias e contábeis. Também as atividades militares, policiais e judiciais, se se viver em um mundo sem guerras e nem crimes, como deve ser. A economia, então, passa ser a verdadeira ciência da produção e distribuição de bens, sem a interveniência do dinheiro, nem tampouco de nenhum sistema de trocas. A extinção da competitividade não extingue o progresso. Pelo contrário. A mudança do espírito competitivo para o espírito colaborativo, em que todos trabalham, uns para o bem dos outros, aí é que propiciará um aumento maior ainda do progresso. Note que esse estágio elevado de civilização requer uma mudança completa na cosmovisão das pessoas. Portanto a anarquia é uma situação que só pode ser alcançada no momento em que ela surgir espontaneamente como um desabrochar interior das pessoas. As pessoas vão fazer tudo o que é preciso, na mais perfeita ordem, porque é o que querem e não porque isso seja determinado por ninguém e nem cobrado por nada. Assim a anarquia ainda levará alguns séculos para ser estabelecida no mundo. Mas, sem dúvida, é o modelo perfeito de sociedade e é uma tendência que vem se estabelecendo nos últimos milênios, como se pode observar pela evolução social da humanidade, cada vez mais democrática e libertária. Tal tendência vai sendo cada vez mais concretizada, por exemplo com a comunidade europeia, com os softwares livres, como o cooperativismo, com os trabalhos em mutirões. Isso tem que ser incentivado cada vez mais. Trabalhar de graça, sem nenhum envolvimento de dinheiro, sem formalização, sem impostos. De forma espontânea e livre. Fazer as coisas à revelia dos governos, até que eles sejam extintos por falta completa de necessidade. Que a polícia seja extinta por falta de serviço. Que as penitenciárias sejam fechadas por falta de presos. Porque ninguém comete crime nenhum. Isso é um processo educativo, em termos de ética e econômico, em termos de prosperidade generalizada.

Em uma sociedade anárquica quem iria produzir os bens materiais de consumo?‎

As mesmas pessoas que os produzem em uma sociedade crática e de economia monetária. Todas as atividades, exceto as desnecessárias para a vida, continuam a ser exercidas normalmente. Só que sem governo e sem dinheiro. Todo mundo trabalha de graça e tem tudo de graça, de uma forma completamente coletivizada e ordeira, sem propriedade privada. Mas não é como no socialismo, em que todos são empregados do governo. Ninguém e empregado de ninguém. Não existem patrões e nem empregados. Só trabalhadores. Tudo é de todo mundo e nada é de ninguém.

Eu votaria a favor de roupas novas para os padres, eles usam sua batina para abusarem de crianças embaixo delas ...‎

Eu votaria a favor da extinção de padres, pastores, rabinos, monges, frades, freiras, imans, lamas, brâmanes e sacerdotes em geral. Acho que nenhuma religião precisa de sacerdotes. As próprias pessoas é que devem fazer os seus serviços, sem dedicação profissional. Melhor ainda seria acabar com as religiões.

Ernesto, como ter sucesso com as mulheres?‎

Sendo você mesmo. Isto é, não encenando nada. Não fazendo uso de nenhum artifício. Mostrando sua própria personalidade, suas próprias qualidades. Se você não for autenticamente uma pessoa cativante, não adianta. Fingir não resolve.

Professor, estou apaixonado por uma garota que tem namorado, mas diz que não gosta dele e está com ele por puro comodismo. Eu já explicitei pra ela meus sentimentos e ela parece interessar por mim, apesar de não deixar explícito. Acha que devo fazê-la trair o namorado comigo ou esperá-la terminar?‎

No meu entendimento acho que você deveria convencê-la a falar com ele logo. Se você admitirem a possibilidade de uma "menage a trois", que ela fique com os dois, desde que todos estejam de acordo com isso e não haja traição. Caso contrário ela teria que fazer uma escolha. Acho que seria bom pressionar para uma solução, uma vez que o imbróglio não é bom para ninguém.

Caro Ernesto Será que o que estava contido no ponto inicial era toda a matéria e energia de um universo “‘anterior” no tempo, como se hoje um buraco negro sugasse tudo que existe até chegar a um ponto insustentável, um estado de concentração tão intenso que causaria uma espécie de colapso repulsivo?‎

Essa é uma possibilidade. que, todavia, está sendo descartadas pelas mais recentes medidas observacionais. Nas soluções cosmológicas das Equações de Einstein da Relatividade Geral, se apresentam três casos: Universo fechado, Universo aberto e Universo limite entre os dois. Tais situações se prendem à comparação entre os parâmetros da aceleração da expansão do espaço e da densidade de massa-energia. Se o primeiro prevalecer, o Universo será aberto e se expandirá indefinidamente. Se o segundo prevalecer o Universo será fechado e se expandirá até um máximo, a partir do qual se contrairá, voltando às condições iniciais de extrema densidade. A terceira possibilidade é o limite de igualdade dos parâmetros, na qual o Universo será aberto e a expansão indefinida também, mas com um comportamento assintótico para uma aceleração nula da expansão. No caso do Universo fechado, a solução para o fator de escala do Universo é uma curva cicloidal que pode ser entendida como se repetindo aquém e além dos pontos de singularidade, em um número infinito de ciclos. Isto é, haveria infinitos Big Bangs, cada um logo após um Big Crunch, de modo que o Universo seria eterno, inclusive para o passado. As mais recentes medidas observacionais desses parâmetros, contudo, indicam que o Universo seja aberto e sem ciclos, expandindo-se indefinidamente, chegando ao Big Rip, em que as quantizações dos campos de matéria em partículas (quarks e léptons) seriam destruídas pela expansão do espaço, acabando com a matéria e deixando o Universo preenchido apenas por campo, já que a radiação também atingiria frequência zero e comprimento de onda infinito. Essa solução não admite ciclos, sendo o Big Bang único e não havendo Universo antes dele (nem sequer "antes"). Se o caso cíclico fosse o que estivesse ocorrendo, ainda estaríamos na fase de expansão e o ciclo demoraria, talvez, várias dezenas de bilhões de anos. Mas, não se tem nenhuma ideia de como seria o processo de passagem do Big Crunch para o Big Bang. O caso que se apresenta como sendo a realidade, também não possui uma explicação para o surgimento do conteúdo, cujo espaço continente passou a se expandir.

LinkWithin

Related Posts with Thumbnails